Pharm PrepU Final Exam Practice

अब Quizwiz के साथ अपने होमवर्क और परीक्षाओं को एस करें!

A nurse is caring for a client daignosed with a serious gram-negative infection prescribed amikacin. Which action should the nurse prioritize while caring for this client? Monitor for constipation Give the drug for no longer than 7 days Do renal tests daily Assess vision daily

Do renal tests daily Explanation: The potential for nephrotoxicity and ototoxicity with amikacin is very high, so the drug is used only as long as absolutely necessary. The increased risk of nephrotoxicity requires daily renal tests to monitor for this adverse effect. Another potential is ototoxicity, so monitoring hearing would be more of a priority than assessing the vision daily. Other adverse effects of amikacin are nausea, vomiting, and diarrhea, not constipation.

A client has edema of the lower extremities with crackles in the lung bases. What diuretic is most likely to be prescribed for a client assessed with lower extremity edema and bilateral lung crackles? Spironolactone Mannitol Hydrochlorothiazide Furosemide

Furosemide Acute pulmonary edema is an indication for the use of furosemide. Hydrochlorothiazide, spironolactone, and mannitol are not used for this purpose, largely due to their slower onset of action.

A client presents at the clinic with a dry, nonproductive cough. The client is diagnosed with bronchitis, and it is determined that they will need help thinning sputum so that the cough can become productive. What does the nurse expect will be prescribed for the client? Aspirin Guaifenesin Codeine Dextromethorphan

Guaifenesin Expectorants, such as guaifenesin, are agents given orally to liquefy respiratory secretions and allow for easier removal. Dextromethorphan and codeine are antitussives used to suppress coughing.

The client has been started on an ACE inhibitor for hypertension. The client also takes spironolactone (Aldactone) daily. The nurse would evaluate the client for which of the following? Hyperkalemia Hypokalemia Hypocalcemia Hypercalcemia

Hyperkalemia When ACE inhibitors are combined with potassium-sparing diuretics, the client is at risk for elevated potassium levels. Therefore, hyperkalemia, not hypokalemia, is the risk. Calcium levels are not affected.

Which is an effect of epinephrine? Peripheral vasodilation Increased heart rate Bronchoconstriction Decreased blood pressure

Increased heart rate Epinephrine increases heart rate and acts as a bronchodilator. It has peripheral vasoconstrictive effects and increases blood pressure.

For a client diagnosed with Parkinson's and narrow angle glaucoma, what medication would be contraindicated? Amantadine Dilantin Neurontin Levodopa

Levodopa Levodopa is contraindicated in clients with known hypersensitivity to the drug. Because levodopa can dilate pupils and raise intraocular pressure, it is contraindicated in narrow-angle glaucoma (because it increases intraocular pressure).

A client is experiencing status epilepticus. Which medication should the nurse prepare to administer? Clonazepam (Klonopin) Valproic acid (Depakote) Diazepam (Valium) Lorazepam (Ativan)

Lorazepam (Ativan) Lorazepam (Ativan) is the drug of choice for status epilepticus. Phenytoin (Dilantin) many also be given to continue longer control of the seizure activity.

A nurse is helping with the discharge of a 92-year-old female client who has been hospitalized for a serious upper urinary tract infection. The client will continue taking SMZ-TMP at home. She is to be discharged into her daughter's care. The nurse will instruct the daughter to watch for which common adverse effect(s) of the drug? Nausea and vomiting Jaundice Photosensitivity reactions Urticaria

Nausea and vomiting Explanation: The nurse should instruct the daughter to watch for nausea and vomiting, which are two common adverse effects of SMZ-TMP, along with diarrhea. Urticaria, photosensitivity reactions, and jaundice are not identified adverse effects of SMZ-TMP.

A nurse is caring for a client receiving diuretic therapy for the treatment of ascites due to malignancy. The nurse suspects that the client is developing hypomagnesemia based on which of the following? Select all that apply. Diarrhea Tachycardia Paresthesias Abdominal distress Tremors

Paresthesias Tachycardia Tremors The symptoms of hypomagnesemia include paresthesias, tachycardia and tremors, leg and foot cramps, hypertension, neuromuscular irritability, hyperactive deep tendon reflexes, confusion, and visual or auditory hallucinations, which need to be monitored. Diarrhea and abdominal distress are the symptoms of hyperkalemia, and so the nurse need not monitor these symptoms in a client with hypomagnesemia.

The nurse is teaching a client about his prescription for a macrolide antibiotic. What should the nurse teach the client to report? Missed doses Lessening of symptoms of infection Increased urinary output Recurring symptoms of infection

Recurring symptoms of infection Explanation: Clients taking antibiotics should be instructed to report symptoms of infection that recur or develop during therapy. Such symptoms can indicate ineffective therapy or development of a new infection with antibiotic-resistant bacteria or fungi.

The nurse is evaluating the education of a client that uses albuterol for an acute asthma attack. The nurse knows that the lesson has been effective when the client states that albuterol is which of the following types of medication? LABA SABA Antiasthma Leukotriene modifier

SABA Albuterol is a short-acting beta-2 agonist (SABA). It is used to treat and prevent bronchospasm.

A 35-year-old female has been recently diagnosed with hyperthyroidism. Her health care provider has prescribed propranolol (Inderal) for what effect? To prevent respiratory depression To decrease heart rate To decrease systolic blood pressure To promote bronchodilation

To decrease heart rate Beta blockers are used to decrease heart rate, decrease cardiac output, and decrease tremors in hyperthyroidism. They are not used to promote bronchodilation, prevent respiratory depression, or to decrease systolic blood pressure.

A 75-year-old male with no history of hypertension has been prescribed nadolol. The nurse knows that the client is likely taking the drug to treat: angina. glaucoma. benign prostatic hyperplasia. asthma.

angina Nadolol is a beta blocker used in the treatment of hypertension and angina.

A nurse is administering an antitubercular drug that exhibits bactericidal activity. Which drug will the nurse administer? ethambutol pyrazinamide rifampin isoniazid

isoniazid Only isoniazid is bactericidal; most other antitubercular drugs such as ethambutol, pyrazinamide, and rifampin are bacteriostatic.

A nurse monitors a client closely for anaphylactic reactions secondary to penicillin therapy. The nurse understands that this occurs more commonly after which route of administration? parenteral oral topical transdermal

parenteral Explanation: Anaphylactic reactions, although possible after oral administration, occur more commonly after parenteral administration. Penicillins are neither administered topically nor transdermally.

A client with constipation is prescribed psyllium. The client asks the nurse how the client should take the medication. What is the nurse's best response? "Drink 4 ounces of soda a day. " "Mix the medication with grapefruit juice and water." "Take this medication with 8 ounces of water." "You should sprinkle the medication on the food."

"Take this medication with 8 ounces of water." Explanation: The client should be instructed to take the psyllium with at least 8 ounces of water or another liquid. The medication is not sprinkled on food and does not need to be mixed with grapefruit juice and water. The nurse should encourage an overall intake of fluids to improve bowel regularity, but not soda.

A client has been prescribed labetalol. What assessment evaluates the effectiveness of the medication? Client no longer reports chest pain on exertion. A decrease from baseline blood pressure. ECG results show no cardiac arrhythmias. Client denies any effects associated with orthostatic hypotension

A decrease from baseline blood pressure. Labetalol is an adrenergic blocking agent prescribed to lower blood pressure. Monitor patient response to the drug by documenting improvement (lowering) in blood pressure. Cardiac arrhythmias and orthostatic hypotension are side effectives of the medication. Abrupt withdrawal of the medication can induce an MI with the associated chest pain.

Which client is most likely to benefit from the administration of an adrenergic agonist? A woman who has been admitted with a suspected ischemic stroke A woman who is in labor and may require a cesarean section A man who has a diagnosis of primary hypertension A man who is in cardiogenic shock following a myocardial infarction

A man who is in cardiogenic shock following a myocardial infarction One of the most frequent indications for adrenergic agonist drugs is shock. Their use is contraindicated in labor, and they would exacerbate hypertension.

A nurse is caring for a 39-year-old client who is taking INH, rifampin, and pyrazinamide. The client reports that her urine is red. What is the most likely cause of this discoloration? Adverse effect of INH Adverse effect of rifampin Hematuria Interaction between INH and rifampin

Adverse effect of rifampin Rifampin causes a harmless red-orange discoloration of urine, tears, saliva, and other body secretions.

The nurse is caring for a client who has developed leukopenia a result of long term sulfonamide therapy. What is the most important nursing intervention in light of this client's condition? Do not administer any intramuscular medications. Handle the client's extremities with care to prevent injury. All individuals in contact with the client must perform frequent hand hygiene. Encourage the client to use a soft toothbrush.

All individuals in contact with the client must perform frequent hand hygiene. Explanation: Leukopenia is a decreased level of white blood cells, and the client is not protected adequately from infection. Therefore, frequent hand hygiene is vital to protect the client from infection. The other interventions are to protect the client who has thrombocytopenia, which is decreased platelets.

A common drug regimen for eradication of H. pylori includes a proton pump inhibitor (PPI) and two antibiotics. What is one of the preferred antibiotics? Ketoconazole Gentamicin Vancomycin Amoxicillin

Amoxicillin Explanation: The treatment of choice for H. pylori infection is a PPI and clarithromycin plus either amoxicillin or metronidazole.

Which condition is treated with an adrenergic agonist? Hypertension Renal failure Anaphylaxis Tachycardia

Anaphylaxis Epinephrine, a prototype adrenergic agonist, is used to relieve the acute bronchospasm and laryngeal edema of anaphylactic shock. Since adrenergic agonists act as cardiac stimulants, some versions may be used to treat severe bradycardia or hypotension.

Propranolol (Inderal) is used extensively as treatment for what condition? Cluster headaches Prevent first MI Hypotension Angina

Angina The beta-adrenergic blocking agents are used to treat cardiovascular problems (hypertension, angina, migraine headaches) and to prevent reinfarction after MI. These drugs are widely used today and include: carteolol (Cartrol), nadolol (Corgard), nebivolol (Bystolic), penbutolol (Levator), pindolol (Visken), propranolol (Inderal), sotalol (Betapace, Betapace AF), and timolol (Blocadren, Timoptic). The prototype drug, propranolol, was in fact the most prescribed drug in the country in the 1980s. Propranolol does not prevent first MIs and it is not used for hypotension or cluster headaches.

A 25-year-old female client who presents at the clinic with vaginal discharge and discomfort is diagnosed with chlamydia. The nurse knows that the drug of choice to treat this infection is doxycycline. What would be a priority assessment for this client before beginning the medication? Asking the client if she has had an allergic reaction to a penicillin Measuring the client's blood pressure Asking the client if she is allergic to sulfonamides Asking the client if she is pregnant

Asking the client if she is pregnant Explanation: Tetracyclines, such as doxycycline, are contraindicated in pregnant women and in children under 8 years of age. In the fetus and young children, the drug can inhibit bone growth and the development of enamel in the teeth.

Which of the following would a nurse identify as being classified as a macrolide? Gentamicin Cephalexin Doxycycline Azithromycin SUBMIT ANSWER

Azithromycin Explanation: Azithromycin is a macrolide. Gentamicin is an aminoglycoside. Doxycycline is a tetracycline. Cephalexin is a cephalosporin.

A nurse is treating a patient who is receiving carbamazepine for a seizure disorder. The nurse knows that carbamazepine is contraindicated in patients with which diseases or disorders? Diabetes Allergy to sulfonamides Bipolar disorder Bone marrow depression

Bone marrow depression Carbamazepine is contraindicated in patients with previous bone marrow depression.

A 54-year-old client has been prescribed sublingual nitroglycerin. After the nurse has finished teaching the client about the medication, what statement could the client make to suggest that the client understands proper self-administration? "I should take as many tablets as I need to make the pain go away." "I should only take the medication if the pain becomes severe." "I should take three tablets as soon as I start to feel chest pain." "I can take up to three tablets at five-minute intervals."

"I can take up to three tablets at five-minute intervals." Clients may take up to three doses of sublingual nitroglycerin at five-minute intervals to relieve anginal chest pain. They should take as few doses as necessary to relieve pain. Administration should begin at the first sign of pain.

A 75-year-old male client was admitted to the unit with angina. The client was started on nadolol . The client asks why this medication was prescribed since the client does not have high blood pressure. What is the nurse's best response? "This drug will prevent you from developing glaucoma." "Some beta blockers are approved as anti-anginal agents." "This medication will reduce benign prostatic hypertrophy as well as treat heart failure." "This medication will prevent blood pressure problems later on."

"Some beta blockers are approved as anti-anginal agents." Decreased heart rate, contractility, and excitability, as well as a membrane-stabilizing effect, lead to a decrease in arrhythmias, a decreased cardiac workload, and decreased oxygen consumption. The juxtaglomerular cells are not stimulated to release renin, which further decreases the blood pressure. These effects are useful in treating hypertension and chronic angina and can help to prevent reinfarction after an MI by decreasing cardiac workload and oxygen consumption. Corgard will not prevent blood pressure problems in the future or prevent you from developing glaucoma. Corgard is not used to treat BPH.

The neurological nurse cares for several clients who have seizure disorders. Which client should the nurse monitor most closely for indications of drug dependence? A client receiving IV phenytoin to prevent seizures post-neurosurgery A client receiving ethosuximide for the prevention of absence seizures A client with a history of myoclonic seizures who takes valproic acid A client with a history of tonic-clonic seizures who takes phenobarbital

A client with a history of tonic-clonic seizures who takes phenobarbital Barbiturates, such as phenobarbital, have an associated risk for dependence. This is not true of the other listed drugs.

Your client is being evaluated for diabetes. He asks you what the blood glucose results mean. Your best response is: A fasting blood sugar result of 126 mg/dL or more on two separate occasions is diagnostic of diabetes. A fasting blood sugar result of 100 mg/dL or more on two separate occasions is diagnostic of diabetes. Two AIC results of 6 or more is diagnostic of diabetes. A fasting blood sugar result of 100 mg/dL or more and an AIC of 6 on two separate occasions is diagnostic of diabetes.

A fasting blood sugar result of 126 mg/dL or more on two separate occasions is diagnostic of diabetes. Explanation: A major clinical manifestation of hyperglycemia is fasting blood glucose levels exceeding 126 mg/dL. An individual with a fasting blood glucose level between 100 mg/dL to 125 mg/dL is said to have impaired fasting glucose or prediabetes.

A nursing instructor is teaching students about miscellaneous drugs that work by inhibiting cell wall synthesis. These include which medications? Select all that apply. vancomycin fluoroquinolones monobactams spectinomycin carbapenems

A nursing instructor is teaching students about miscellaneous drugs that work by inhibiting cell wall synthesis. These include which medications? Select all that apply. carbapenems vancomycin monobactams

A client presents to the clinic with uncomplicated urinary tract infection. The client does not have any know drug allergies and is not taking any home medications. Which medication would most likely be given to this client? A macrolide A second-generation fluoroquinolone A ketolide A lipopeptide

A second-generation fluoroquinolone Explanation: First- and second-generation fluoroquinolones are used primarily to treat uncomplicated urinary tract infections because of their ability to concentrate in the urine. Macrolides are generally not used to treat urinary tract infections because they tend to have higher concentrations in the lungs and skin. And thus, they are used for respiratory tract infections. Lipopeptides are primarily used to treat endocarditis and skin infections. Ketolides are designed to combat respiratory tract pathogens that have acquired resistance to macrolides.

When caring for a client diagnosed with a peptic ulcer, the nurse administers omeprazole and amoxicillin. What diagnostic finding indicates therapeutic effects of this drug regimen? Absence of dyspepsia Absence of gastresophageal reflux Increased appetite and absence of nausea Absence of Helicobacter pylori

Absence of Helicobacter pylori Gastric acid pump or proton pump inhibitors are recommended in combination with amoxicillin and clarithromycin for the treatment of H. pylori infection. Eradication of this bacterium would be the priority outcome, due to the fact that the client is taking an antibiotic.

A nurse demonstrates understanding of diuretics when identifying which medication as exerting its effect by inhibiting the enzyme carbonic anhydrase? Spironolactone Acetazolamide Furosemide Hydrochlorothiazide

Acetazolamide Acetazolamide is a carbonic anhydrase inhibitor that exerts its effect by inhibiting the enzyme carbonic anhydrase. Furosemide is a loop diuretic. Hydrochlorothiazide is a thiazide diuretic. Spironolactone is a potassium-sparing diuretic.

Which of the following clients should not be prescribed fluoroquinolones? Female 12 years of age with a respiratory infection Male 54 years of age with prostatitis Female 79 years of age with a urinary tract infection Male 19 years of age with pneumonia

Female 12 years of age with a respiratory infection Explanation: Fluoroquinolones are contraindicated in children under 18 years of age. Fluoroquinolones are used to treat respiratory infections and prostatitis.

The pharmacology instructor is discussing ways to decrease adverse effects on clients taking aminoglycosides. Which would the instructor include? Assessing liver function weekly Increasing dietary sodium Keeping the client well hydrated Decreasing dietary potassium

Keeping the client well hydrated Explanation: Aminoglycosides are nephrotoxic. Keeping the client adequately hydrated (by increasing daily fluid intake to two to three liters) helps decrease the risk of adverse effects related to renal function. Renal function tests should be performed regularly during aminoglycoside therapy.

A client comes to the emergency department. The client is experiencing continuous seizure activity without any interruptions. Which anticonvulsant would the nurse anticipate that the primary health care provider would most likely prescribe initially? Valproic acid Clorazepate Phenytoin Lorazepam Fosphenytoin

Lorazepam Lorazepam (Ativan) is the drug of choice for this condition. However, because the effects of lorazepam last less than 1 hour, longer-lasting anticonvulsants such as phenytoin or fosphenytoin may be given to continue to control the seizure activity. Valproic acid is used to treat epilepsy, migraine headache, and mania. Clorazepate is used to treat focal seizures, anxiety disorders, and alcohol withdrawal.

A nurse is preparing to administer an anticonvulsant that produces its effects by stabilizing the hyperexcitability postsynaptically in the motor cortex of the brain. Which medication would the nurse administer? Lorazepam Valproic acid Ethosuximide Phenytoin

Phenytoin Hydantoins like phenytoin elicit their effects by stabilizing the hyperexcitability postsynaptically in the motor cortex of the brain. Valproic acid increases the levels of GABA to help stabilize the membranes. Ethosuximide depresses the motor cortex, creating a higher threshold before the nerves react to the convulsive stimulus. Lorazepam elevates the seizure threshold by decreasing postsynaptic excitation.

The nurse is preparing to administer a client's scheduled dose of Sinemet. What is the nurse's rationale for administering levodopa with carbidopa? Carbidopa makes the blood-brain barrier temporarily permeable. Carbidopa potentiates the action of levodopa to prevent nerve cell degeneration. Carbidopa prevents excessive amounts of levodopa from being excreted by the kidneys and in feces The combination allows a lower dose of levodopa than if levodopa were administered alone.

The combination allows a lower dose of levodopa than if levodopa were administered alone. Carbidopa decreases the amount of levodopa needed to reach a therapeutic level in the brain so that the dosage of levodopa can be decreased, which reduces the incidence of adverse side effects. When levodopa is used in combination with carbidopa, the enzyme dopa is inhibited in the periphery, diminishing the metabolism of levodopa in the GI tract and in peripheral tissues, thereby leading to higher levels crossing the blood-brain barrier.

The nurse is administering epinephrine IV push. The nurse checks for patency of the IV prior to administration of the drug. The nurse realizes that if the IV is infiltrated, the medication will result in tissue extravasation. What is the reason for such an occurrence? The medication causes swelling within the tissues, impairing blood flow to the site. The medication causes an allergic response at the site of infiltration, leading to swelling and erythema. The medication causes venous congestion and pooling of blood in the tissues. The medication causes vasoconstriction, limiting the blood flow to the area.

The medication causes vasoconstriction, limiting the blood flow to the area. Extravasation can occur during parenteral administration; necrosis may result because of epinephrine's potent vasoconstrictive properties.

A client has been diagnosed with Parkinson's disease and the nurse is providing health education. The client asks the nurse, "Why can't I take a dopamine pill if that's what is lacking?" When responding, the nurse should address what topic? The differences in anatomy and physiology between the central nervous system and the peripheral nervous system The role of acetylcholinesterase The structure and function of the blood-brain barrier The functioning of the sodium-potassium pump

The structure and function of the blood-brain barrier Dopamine cannot cross the blood-brain barrier, so exogenous supplements would have no therapeutic benefit. This is unrelated to the functions of acetylcholinesterase, sodium-potassium pumps or the CNS and PNS.

A client is taking levodopa. What should the nurse caution the client to avoid? Vitamin B6 Analgesics St. John's wort Aged cheese

Vitamin B6 Vitamin B6 interacts with levodopa decreasing its effectiveness. St. John's wort is not known to interact with levodopa, but does interact with rasagiline. Aged cheese contains tyramine and should be avoided if the client is taking rasagiline, but there is no caution associated with levodopa. Analgesics are not known to interact with levodopa.

The nurse is caring for a 27-year-old female client who has just been prescribed misoprostol. What is a priority teaching point for this client? You will need to use a barrier-type contraceptive Adverse effects include nausea and diarrhea It protects the lining of the stomach Do not take NSAIDs with this drug

You will need to use a barrier-type contraceptive Misoprostol is contraindicated during pregnancy because it is an abortifacient. Women of childbearing age who use misoprostol should be advised to use barrier-type contraceptives. All other options are correct but are not a priority for this client because they are less of a safety risk.

What are the classic symptoms associated with Parkinson's disease? Select all that apply. urinary retention resting tremors muscle rigidity worsening eyesight bradykinesia

bradykinesia muscle rigidity resting tremors Classic symptoms of Parkinson's disease include resting tremor, bradykinesia, rigidity, and postural instability.

A 72-year-old man should be aware that the adverse effects of centrally acting anticholinergic drugs include: gout. diabetes mellitus. confusion. sedation.

confusion Anticholinergic drugs may cause agitation, mental confusion, hallucinations, and psychosis in the older adult.

Penicillins are most effective when used to treat what type of microorganisms? gram-negative bacilli gram-negative bacteria gram-positive bacteria fungal infections SUBMIT ANSWER

gram-positive bacteria Explanation: Clinical indications for use of penicillins include bacterial infections caused by susceptible microorganisms. As a class, penicillins usually are more effective in infections caused by gram-positive bacteria than those caused by gram-negative bacteria. However, their clinical uses vary significantly according to the subgroup or individual drug and microbial patterns of resistance. This information makes the remaining options incorrect. Bacilli are a type of bacteria.

A client diagnosed with Parkinson's disease is being treated with tolcapone. When reviewing the client's medication history, the nurse should confirm that the client is concurrently taking what other drug? atropine benztropine levodopa/carbidopa ipratropium

levodopa/carbidopa It is necessary to administer tolcapone in conjunction with levodopa/carbidopa and to monitor the client's response to the medication. Ipratropium, atropine, and benztropine are not indicated.

A nursing student studying pharmacology is focusing on drugs used to treat tuberculosis (TB). This student correctly identifies the classifications of antitubercular drugs as which of the following? primary and secondary antibacterial and antifungal primary and tertiary first and second generation

primary and secondary When classifying antitubercular drugs there are primary (first-line) and secondary (second-line) drugs. Primary drugs provide the foundation for treatment. Secondary drugs are less effective and more toxic than primary drugs. First- and second-generation drugs are two types of cephalosporins. Primary and tertiary refers to types of care. Antibacterial and antifungal also are not correct.

The nurse is caring for a public speaker who has developed severe stage fright that is preventing them from working. What drug should the nurse suspect may be prescribed for this client? nadolol carteolol propranolol nebivolol

propranolol One of the indications for use of propranolol is prevention of stage fright, which is a sympathetic stress reaction to a particular situation. None of the other options are indicated for this use.

The goal of therapy for a client taking antihypertensive medication is to maintain: a fluid volume balance. compliance. homeostasis. the blood pressure within normal limits.

the blood pressure within normal limits. Helping the client to maintain the blood pressure within normal limits is the goal when drug therapy is instituted.

Which adrenergic receptor is responsible for vasodilation of peripheral vessels and bronchodilation? β2 receptors β1 receptors α2 receptors α1 receptors

β2 receptors Vasodilation of peripheral vessels and bronchodilation are the result of stimulation of β2 receptors.

A nurse is teaching a client about omeprazole, which has been prescribed as part of a regimen to treat an H. pylori infection. What statement, made by the client, suggests that the client understands proper use of the drug? "I need to swallow the drug whole." "I'll have to stop using antacids." "I should crush the medication and mix it into apple sauce." "It is important that I take the drug after each meal."

"I need to swallow the drug whole." Omeprazole, a proton pump inhibitor, must be swallowed whole; it should not be cut, crushed or chewed. The drug should be taken approximately an hour before a meal, not after a meal. Concomitant use of antacids is common, though the health care provider may instruct the client to take the two medications at different times during the day.

The nurse is discharging a 35-year-old client with diabetes who has been prescribed an adrenergic blocking agent. What is the priority teaching point for the nurse to discuss with this client? "Document signs and symptoms of hyperglycemia and hypoglycemia" "Monitor blood glucose levels closely and report any instability" "Reduce carbohydrate intake more than usual while taking the new drug" "Increase insulin dosage to compensate for the drug's effect in increasing blood sugar"

"Monitor blood glucose levels closely and report any instability" It is important for the client to be instructed to monitor blood sugar levels more frequently because adrenergic blocking agents mask the normal hypo- and hyperglycemic manifestations that normally alert clients such as sweating, feeling tense, increased heart rate, and rapid breathing. There is no need to change the diet or the diabetic medications. There may be no signs and symptoms to record because they are blocked by the adrenergic blocker.

A client has been on sulfonamide therapy for the last six weeks. What client report may cause the health care provider to discontinue the sulfonamide? Loose stools for the last two days Unable to eat spicy foods 10 lb weight loss Decreased appetite

10 lb weight loss Explanation: GI side effects with sulfonamide therapy may be mild, such as decreased appetite, inability to eat spicy foods (due to stomatitis), or loose stools for two days. But the drug may need to be discontinued for more serious problems such as pronounced weight loss.

A patient is receiving acetazolamide in a sustained release form. The nurse would anticipate the onset of drug action in approximately which time frame? 1 hour 2 hours 3 hours 4 hours

2 hours Acetazolamide in sustained release form has an onset of action of 2 hours.

An older adult has been prescribed midodrine. What is the most important nursing action to include in the plan of care for this client? Ensure the client has ready access to the call light. Auscultate the client's apical heart rate hourly. Establish continuous pulse oximetry following the first dose. Assess positional blood pressures.

Assess positional blood pressures. Midodrine is an oral drug used to treat orthostatic hypotension in clients who do not respond to traditional therapy. It activates alpha-adrenergic receptors, leading to peripheral vasoconstriction and an increase in vascular tone and blood pressure. This effect can cause serious supine hypertension. Patients should be monitored in the standing, sitting, and supine positions to determine whether this will be a problem. It is also important to monitor heart rate and oxygenation in this client. However, assessing for supine hypertension would pose the greatest threat to the client and would take priority. Ensuring call light access is an important safety measure but is not directly related to the administration of midodrine.

The nurse provides discharge teaching for a female client who will receive a prescription for cefaclor. What important information should the nurse provide this client? Avoid alcohol until 72 hours after stopping this medication. Avoid grapefruit juice when taking this medication to prevent adverse effects. Vaginal itching will go away after the drug is discontinued. Monitor for yellowing of the skin or eyes and call the health care provider if it occurs.

Avoid alcohol until 72 hours after stopping this medication. Explanation: Patients should be taught to avoid alcohol for up to 72 hours after discontinuing cefaclor to prevent a disulfiram-like reaction that results in unpleasant symptoms such as flushing, throbbing headache, nausea and vomiting, chest pain, palpitations, dyspnea, syncope, vertigo, blurred vision, and in extreme reactions, cardiovascular collapse, convulsions, or even death. Genital itching in women indicates the possibility of a superinfection, and the client should see her healthcare provider. Liver damage, indicated by jaundice, is not a likely adverse effect with this drug. There is no need to avoid grapefruit juice.

A client is experiencing allergy symptoms after being exposed to environmental dust. This reaction involves the action of histamine, which is released by what cells? CD4 T cells Basophils Lymphocytes Platelets

Basophils Histamine is discharged from mast cells and basophils in response to certain stimuli, such as allergic reactions, cellular injury, and extreme cold. Lymphocytes and platelets do not release histamine. CD4 cells (often called T cells or T-helper cells) are a type of white blood cells that play a major role in protecting the body from infection.

What is the most common first-line therapy for relief of an acute asthma attack? Beta2-adrenergic agonist Leukotriene modifier Inhaled steroid Xanthine

Beta2-adrenergic agonist A client experiencing an acute asthma attack should be administered a beta2-adrenergic agonist. The client can receive an inhaled steroid, but it is not the first-line therapy. Leukotriene modifiers are used for maintenance in asthma, not during acute exacerbation. Xanthines are not the drug of choice in acute asthma attack.

A client has come to get lab results at the cardiology clinic. The nurse notes that the digoxin level is elevated. The nurse knows that what other medication may be responsible for the increase in digoxin level? Zantac Carvedilol Prozac Ampicillin

Carvedilol When taking both carvedilol and digoxin, the client's digoxin level becomes higher and the client is at greater risk of digoxin toxicity. Prozac, Zantac, and ampicillin do not increase a client's digoxin level.

Culture and sensitivity testing of a client's urine sample reveals a bacterium that is susceptible to cephalosporins. What medication would be most likely for the nurse to administer? Ciprofloxacin Neomycin Imipenem-cilastatin Cefaclor

Cefaclor Explanation: Cefaclor is an example of a cephalosporin. Imipenem-cilastatin is an example of a carbapenem. Neomycin is an example of an aminoglycoside. Ciprofloxacin is an example of a fluoroquinolone.

Which drug is a first-generation cephalosporin used as a surgical prophylaxis? Ceftriaxone Cefazolin Cefotetan Cefoxitin

Cefazolin Explanation: Cefazolin is a first-generation cephalosporin. Cefotetan and cefoxitin belong to the second generation, and ceftriaxone is a third-generation cephalosporin.

A group of students are reviewing the various drugs that affect inflammation. The students demonstrate understanding when they identify which agent as a mast cell stabilizer? Cromolyn Montelukast Triamcinolone Calfactant

Cromolyn Cromolyn is a mast cell stabilizer. Montelukast is a leukotriene receptor antagonist. Calfactant is a lung surfactant. Triamcinolone is an inhaled steroid.

The nurse is caring for a client with peripheral edema who has just begun taking a diuretic. What assessments should the nurse use to evaluate the effectiveness of this medication? Select all that apply. Daily weights Chest auscultation Inspection of lower legs and feet Urine output Level of consciousness

Daily weights Inspection of lower legs and feet Urine output Responsiveness to the use of a diuretic can be measured by daily weights, assessment for edema, and increased urine output. No change in level of consciousness is expected. The client has peripheral edema, not pulmonary edema, so chest auscultation would not be a priority assessment.

A nurse is preparing to administer a client's first scheduled dose of tetracycline. The nurse should first ensure that the client has not recently eaten what type of foods? Leafy green vegetables Dairy products Acidic foods Any high-fat foods

Dairy products Explanation: The combination of tetracycline with metallic ions such as aluminum, calcium, iron, or magnesium inhibits tetracycline absorption. It is important not to take tetracycline with dairy products, antacids, or iron supplements. None of other foods present significant amounts of metallic ions.

An older adult has regularly drunk mineral oil to treat occasional episodes of constipation. The nurse should encourage the client to explore other treatments because the use of oral mineral oil can result in which adverse effect? Decreased absorption of drugs and vitamins Edema from changes in intestinal solute content Paralytic ileus Rebound constipation

Decreased absorption of drugs and vitamins Oral use of mineral oil may cause potentially serious adverse effects, including decreased absorption of fat-soluble vitamins and some drugs and lipid pneumonia if aspirated into the lungs. Thus, mineral oil is not an oral laxative of choice in any condition. It is not noted to cause rebound constipation, paralytic ileus, or edema.

Decongestants should be used cautiously in clients with which medical conditions? Select all that apply. Glaucoma Thyroid disease Hypertension Asthma Diabetes

Diabetes Thyroid disease Glaucoma Hypertension Decongestants are used cautiously in clients with the following: thyroid disease, diabetes, cardiovascular disease, prostatic hypertrophy, coronary artery disease, peripheral vascular disease, hypertension, and glaucoma. Clients with asthma benefit from the use of decongestants during episode of upper respiratory infections.

The nurse is preparing a teaching plan for a client who is prescribed ceftriaxone. What should the nurse identify as common adverse effects associated with this drug? Headache and dizziness Diarrhea and nausea Lethargy and paresthesias Superinfections and phlebitis

Diarrhea and nausea Explanation: The most common adverse effects associated with ceftriaxone, a cephalosporin, include those affecting the GI tract, such as nausea, vomiting, diarrhea, abdominal pain, and flatulence. Headache, dizziness, lethargy, and paresthesias can occur as well as superinfections and phlebitis (with IV administration), but these are not the most common.

What drug is a safe and effective calcium channel blocker only if the nurse administers it as a sustained-release or extended-release preparation to treat hypertension? Diltiazem Atenolol Aliskiren Metoprolol

Diltiazem The calcium channel blockers available in immediate-release and sustained-release forms that are used in treating hypertension include amlodipine, felodipine, isradipine, and nicardipine. Other calcium channel blockers are safe and effective for this use only if they are given as sustained-release or extended-release preparations. These include diltiazem, nifedipine, nisoldipine, and verapamil. Aliskiren is a renin inhibitor. Atenolol and metoprolol are beta-blockers, not calcium channel blockers.

A new mother required an episiotomy during the birth of her baby. Two days after delivery, the client is in need of a laxative. What will be the most effective drug for the nurse to administer? Bisacodyl Docusate Castor oil Magnesium sulfate

Docusate Explanation: Docusate is a surfactant laxative that will make expulsion of stool easier in a traumatized body area following an episiotomy. Care must be taken to choose a mild medication that will not enter breast milk and not affect the newborn if the mother is nursing. Docusate is the drug of choice from this list because it is mild and will produce a soft stool and decrease the need to strain. The other options would not be appropriate because they do not soften the stool and are harsher laxatives that can enter breast milk.

The nurse practitioner instructs the client to use over-the-counter ranitidine instead of cimetidine. What risk is decreased when using ranitidine rather than cimetidine? Drug-to-drug interaction Bradycardia Headache Diarrhea

Drug-to-drug interaction Ranitidine causes fewer drug interactions than cimetidine. Ranitidine possesses the following adverse effects: headache, diarrhea, and bradycardia.

The nurse is providing care for a client with Parkinson's disease. Which of the client's medications will most directly affect catecholamine-O-methyl transferase (COMT) function? Benztropine Entacapone Diphenhydramine Selegiline

Entacapone Entacapone inhibits COMT. Selegiline mechanism of action is unknown, but it does irreversibly inhibit monoamine oxidase, not COMT. Benztropine opposes the effects of acetylcholine at receptor sites in the substantia nigra and corpus striatum. Diphenhydramine opposes the effects of acetylcholine at receptor sites in the substantia nigra and corpus striatum.

A client has an upper respiratory infection and is allergic to penicillin. What medication does the nurse expect will be ordered? Ampicillin Ancef Cephalosporin Erythromycin

Erythromycin Explanation: A macrolide can be given safely to a client who is allergic to penicillin. Ampicillin, Ancef, and cephalosporins cannot be given because of the risk of cross-sensitivity.

A client who is a newly diagnosed diabetic is prescribed glyburide. The nurse caring for this client identifies which occurrence is a classic symptom of hyperglycemia? Grand mal seizures Excessive urination Hemiparesis Tachycardia

Excessive urination Explanation: The classic symptoms of hyperglycemia include excessive urination (polyuria) and excessive thirst (polydipsia) caused by the osmotic pull of glucose. Grand mal seizures and hemiparesis are symptoms of nonketotic hyperglycemia, while tachycardia is a symptom of diabetic ketoacidosis.

A nurse is completing an admission assessment of a client who is having diarrhea. The client states that the diarrhea started after "abdominal surgery." The client cannot recall the type of surgery. Which does the nurse suspect as the cause of the diarrhea? Skin infection of the surgical site Surgical repair for colon polyps Liver resection Excision of the small intestine

Excision of the small intestine Explanation: Surgical excision of portions of the intestine, especially the small intestine, can decrease the absorptive area and increase fluidity of stools. Skin infections, surgical repair of colon polyps and liver resection do not cause diarrhea.

A client taking a cephalosporin for a respiratory infection informs the nurse that her tongue feels funny and has white patches on it. She asks the nurse what could be wrong. The nurse informs the client that she has developed which? Adverse drug reaction Fungal superinfection Complication from the respiratory problem Poor dental hygiene

Fungal superinfection Explanation: A fungal superinfection commonly occurs in the mouth, vagina, and anogenital areas. Common signs of an oral fungal infection are creamy, white, lace-like patches on the tongue, mouth, or throat. The other options are not appropriate choices as the causes for these signs and symptoms.

Which type of seizures involves a loss of consciousness? Somatosensory seizures Partial seizures Motor seizures Generalized seizures

Generalized seizures Generalized seizures involve a loss of consciousness.

Which is the best indicator of overall diabetic control? Absence of acetone in the urine Glycosylated hemoglobin levels Fasting blood glucose levels 2-hour postprandial blood glucose levels

Glycosylated hemoglobin levels Explanation: The glycosylated hemoglobin indicates glucose bound to hemoglobin in red blood cells (RBCs) when RBCs are exposed to hyperglycemia. The binding is irreversible and lasts for the lifespan of RBCs (approximately 120 days). The test reflects the average blood sugar level during the previous 2 to 3 months. The goal is usually less than 7% (blood level 0.07). The range for people without diabetes is approximately 4% to 6% (blood level 0.04 to 0.06).

The nurse is caring for a client who is taking an adrenergic bronchodilator. In what disease process should adrenergic bronchodilators be used cautiously? Heart failure Liver failure Respiratory failure Renal failure

Heart failure Adrenergic drugs cause cardiac stimulation and may be contraindicated in heart failure. Patients with liver failure, renal failure, or respiratory failure do not need to use adrenergic bronchodilators cautiously.

The adrenergic branch of the nervous system controls many functions in the body. The nurse knows that which body functions are controlled by the adrenergic branch? (Select all that apply.) Heart rate Gross muscle movement Respiratory rate Muscle blood flow fine muscle movements

Heart rate Respiratory rate Muscle blood flow The adrenergic branch of the nervous system controls heart rate, respiratory rate, and blood flow to the muscles. Muscle movement (fine or gross) is controlled by the somatic nervous system.

The nurse is caring for a neonate of a mother who took a sulfonamide during the last weeks of pregnancy. For which condition will the nurse monitor the neonate? Select all that apply. Hypothermia Hemolytic anemia Thrombocytopenia Jaundice Rash

Hemolytic anemia Jaundice Explanation: If the sulfonamides are given near the end of pregnancy, significantly higher blood levels of the drug may occur, causing hemolytic anemia and jaundice in the neonate. The use of sulfonamides near the end of pregnancy do not cause hypothermia, rash, or thrombocytopenia.

The emergency department nurse is asked to prepare a nitroprusside infusion for a client being brought to the hospital in an ambulance. What is this client's most likely diagnosis? Hypertension associated with diabetic ketoacidosis Hemorrhagic stroke Myocardial infarction accompanied by hypertension Hypertensive crisis

Hypertensive crisis Most of the vasodilators are reserved for use in severe hypertension or hypertensive emergencies. These include hydralazine, minoxidil, and nitroprusside. The presence or absence of a comorbidity such as stroke, diabetic ketoacidosis, or MI does not increase the likelihood of use.

The nurse is preparing to administer a sulfonamide to a client who is a Type 2 diabetic taking an oral hypoglycemic agent. The nurse will monitor the client for what reaction? A)Increased bruising B)Hypoglycemia C)Hyperglycemia D)Elevated temperature

Hypoglycemia Explanation: Diabetic clients who take a sulfonamide may experience a hypoglycemic reaction because the sulfonamides may inhibit the hepatic metabolism of the oral hypoglycemic drugs tolbutamide and chlorpropamide. The nurse would not expect to see increased bruising or an elevated temperature, which may be a reaction related to bone marrow suppression.

The client is given epinephrine for an exacerbation of asthma. What response to the medication would the nurse expect? Bronchodilation and decreased heart rate Decreased inflammation and increased heart rate Increased ease of breathing and increased heart rate Increased heart rate and respiratory rate

Increased ease of breathing and increased heart rate Depending on the location and distribution of receptors, epinephrine exerts a variety of responses in different effector organs and tissues. In the cardiovascular system, epinephrine exerts positive inotropic and chronotropic effects on the myocardium by stimulating beta-1 adrenergic receptors. In the skin and viscera, epinephrine stimulates alpha-adrenergic receptors, causing vasoconstriction and vasodilation in skeletal muscle vessels. The overall effect is to increase systolic pressure and slightly decrease diastolic pressure. In the respiratory system, epinephrine causes bronchodilation by the stimulation of beta-2 adrenergic receptors and is used in this way to treat clients with asthma or to manage anaphylactic shock.

A client arrives at the walk-in clinic reporting vomiting and a burning sensation in the mid-epigastric region. The nurse suspects that the client has peptic ulcer disease. Which is a potential cause of peptic ulcer disease? Decreased stomach acid secretion Overconsumption of spicy foods Infection with H. pylori Excessive worrying

Infection with H. pylori Peptic ulcers can result from cell destruction caused by Helicobacter pylori infection, gastric acid, pepsin, and ingestion of nonsteroidal anti-inflammatory drugs (NSAIDs).

The nurse is administering penicillin to a client who has strep throat. Which of the following statements accurately describe the action of penicillin? It is effective against gram-positive organisms It is not effective against gram-negative organisms It is metabolized in the liver It has many side effects, especially in large doses

It is effective against gram-positive organisms Explanation: Penicillin is most effective against gram-positive organisms, such as streptococci, staphylococci, and pneumococci. It is also active against some gram-negative organisms, such as gonococci and meningococci, and against the organisms that cause syphilis. It is relatively free of side effects, even in larger doses, and is excreted rapidly in the urine.

The nurse is caring for a client diagnosed with Parkinson disease who has been prescribed an anticholinergic drug. What factor most likely prompted the primary care provider to prescribe this drug? The client has exhibited adverse effects of medications. The client's disease has been deemed incurable. The client has developed psychological dependence on dopaminergics. Levodopa has not caused a sufficient therapeutic response.

Levodopa has not caused a sufficient therapeutic response. Although anticholinergics are not as effective as levodopa in the treatment of advancing cases of the disease, they may be useful as adjunctive therapies and for clients who no longer respond to levodopa. They are not given in response to adverse effects. Parkinson disease is never considered curable. Dopaminergics are not associated with psychological dependence.

A patient is admitted to the emergency room with uncontrolled tonic-clonic seizures. What medication is administered intravenously to assist in reducing seizure activity? Ethosuximide (Zarontin) Gabapentin (Neurontin) Lorazepam (Ativan) Phenytoin (Dilantin)

Lorazepam (Ativan) The drug of choice is an intravenous benzodiazepine, usually lorazepam. A systematic review of trials in adult and pediatric patients concluded lorazepam was significantly better than diazepam or phenytoin for the immediate control of status epilepticus. Ethosuximide (Zarontin) is not administered for status epilepticus. Gabapentin (Neurontin) is not administered for status epilepticus.

A client has refused a scheduled dose of metformin, stating that he/she is worried about inducing hypoglycemia because his/her blood glucose level is currently 66 mg/dL (3.66 mmol/L). The nurse should convey what teaching points to the client? Metformin does not cause hypoglycemia. Hypoglycemia is only a risk in clients with type 1 diabetes. Overuse of metformin creates a risk for hyperglycemia, not hypoglycemia. If the client has been taking metformin for more than 3 to 4 weeks, there is no risk for hypoglycemia.

Metformin does not cause hypoglycemia. Explanation: Experts prefer to call metformin an antihyperglycemic rather than a hypoglycemic because it does not cause hypoglycemia, even in large doses, when used alone. This fact about metformin makes the other statements inaccurate.

Numerous residents of a long-term care facility have developed Clostridium difficile-associated diarrhea over the past week. The nurses at this facility would anticipate that many residents would require which medication therapy? Metronidazole Linezolid Chloramphenicol Daptomycin

Metronidazole Explanation: Clinical indications for metronidazole include prevention or treatment of anaerobic bacterial infections (e.g., in colorectal surgery, intra-abdominal infections) and treatment of C. difficile infections associated with pseudomembranous colitis. The other listed antibiotics are not used for the treatment of Clostridium difficile-associated diarrhea.

Epinephrine is contraindicated in clients with which diseases or disorders? Asthma Hypotension Bronchoconstriction Narrow-angle (or closed-angle) glaucoma

Narrow-angle (or closed-angle) glaucoma Epinephrine and other adrenergic drugs are contraindicated in clients with narrow-angle glaucoma (closed-angle glaucoma) because the drugs increase intraocular pressure and mydriasis. Asthma, bronchoconstriction, and hypotension are among the disorders and diseases treated with adrenergic drugs.

When administering an adrenergic drug, a nurse understands that which is the primary neurotransmitter of the sympathetic nervous system? Acetylcholine Norepinephrine Dopamine Serotonin

Norepinephrine Norepinephrine is the primary neurotransmitter of the sympathetic nervous system. Dopamine is a catecholamine neurotransmitter that is released during the body's stress response and includes norepinephrine and epinephrine. In the brain, serotonin acts as a neurotransmitter that is involved in the control of pain perception, the sleep-wake cycle, and mood. Acetylcholine acts as a neurotransmitter between muscle and the nerve endings of the parasympathetic nervous system.

Pseudoephedrine (Sudafed) is prescribed for a client suffering from nasal and sinus congestion. What is the most important assessment the nurse should complete before the client leaves the clinic to purchase the medication? Ask about history of hay fever. Observe for swelling in nasal passages. Ask about nausea and vomiting. Obtain the client's blood pressure.

Obtain the client's blood pressure. Sudafed is a decongestant. Decongestants are used cautiously in clients with a history of hypertension because of the vasoconstrictive effects of the drug. Swelling of nasal passages is a symptom of congestion. A history of hay fever is not a contraindication to taking decongestants. Nausea and vomiting are potential adverse effects but are not as important as assessing blood pressure.

After reviewing the routes of administration for beta-specific adrenergic agonists, a group of students demonstrate the need for additional study when they identify isoproterenol as being administered by which route? Intramuscular Oral Intravenous Subcutaneous

Oral Isoproterenol is administered IV, IM, or subcutaneously. It is not administered orally.

Clients who are taking nifedipine should be monitored for which of the following adverse effects? Peripheral edema Tetany Hyperkalemia Asthma

Peripheral edema Nifedipine is a calcium channel blocker. Of the four choices offered, only peripheral edema is an adverse effect of this drug class.

A client is suspected of having pheochromocytoma. Which agent would the nurse identify as being used to help diagnose this disorder? Phentolamine Labetalol Carvedilol Atenolol

Phentolamine Phentolamine is the drug used to diagnose pheochromocytoma. Labetalol is used to treat the hypertension that is associated with pheochromocytoma. Carvedilol is used to treat hypertension, but not that associated with pheochromocytoma. Atenolol is used to treat myocardial infarction, chronic angina, and hypertension; it is not used to diagnose pheochromocytoma.

The home care nurse is caring for an older adult client who has type 1 diabetes. The client has visual impairment and cannot read the numbers on the syringe when preparing insulin for administration nor afford the cost of prefilled auto syringes. What strategy might the nurse use to help this client comply with insulin needs between visits? Change the client to oral antidiabetics. Ask a neighbor to come over every day to prepare the medication. Have the client use a magnifying glass. Prepare a week's supply of syringes and refrigerate.

Prepare a week's supply of syringes and refrigerate. Explanation: Older adults can have many underlying problems that complicate diabetic therapy. Poor vision and/or coordination may make it difficult to prepare a syringe. A week's supply of syringes can be prepared and refrigerated for the usual dose of insulin. If the client is using insulin, it is most likely because oral antidiabetic medications don't work. A magnifying glass is impractical because drawing up medication requires two hands and a magnifying glass will not help the client to see well enough to be safe. It is a big imposition to expect a neighbor to be constantly available and this would not be the best choice.

A client with peptic ulcer disease is experiencing fewer symptoms because of suppression of hydrochloric acid secretion into the lumen of the stomach. What category of medication is this client most likely receiving? Histamine-2 antagonist Antiseptic agent Proton pump inhibitor Prostaglandin

Proton pump inhibitor Explanation: Proton pump inhibitors suppress the secretion of hydrochloric acid into the lumen of the stomach. Antiseptic agents coat any injured area in the stomach to prevent further injury. H2 antagonists block the release of hydrochloric acid in response to gastrin. Prostaglandins inhibit secretion of gastrin and increase secretion of the mucous lining of the stomach.

A nurse is caring for a client who has been prescribed diphenhydramine for the treatment of Parkinson disease in an older adult. The client reports diplopia and blurred vision. Which intervention would be appropriate? Provide eye protection to the client. Instruct the client to place a cool cloth over eyes. Instruct the client to avoid direct sunlight. Provide proper assistance with ambulation.

Provide proper assistance with ambulation. The nurse should provide proper assistance with ambulation to the client who has developed a visual difficulty. Instructing the client to avoid direct sunlight, providing eye protection to the client, and instructing the client to place a cool cloth over the eyes are not appropriate interventions because they will not help reduce the client's discomfort due to diplopia, nor will they minimize the symptoms.

A client with a history of peptic ulcer disease has responded well to treatment with a histamine-2 antagonist. What effect will this drug have? Coat and protect the stomach lining Increase the pH of secreted hydrochloric acid. Reduce the quantity of hydrochloric acid secreted. Destroy Helicobacter pylori.

Reduce the quantity of hydrochloric acid secreted. Histamine-2 antagonists are administered to reduce the amount of hydrochloric acid secreted in the stomach, which helps to prevent peptic ulcer disease. H2 antagonists do not act as an antibiotic to kill bacteria (i.e., H. pylori) coat and protect the stomach lining like sucralfate, or increase the pH of the secreted hydrochloric acid.

A client is started on sulfamethoxazole-trimethoprim for a urinary infection. What adverse effect should the nurse assess with this client? Renal damage Bone marrow depression Congestive heart failure Liver toxicity

Renal damage Explanation: Renal impairment is a possible side-effect to the administration of SMZ-TMP. It does not cause liver toxicity, bone marrow depression, or congestive heart failure.

A client, being treated in the intensive care unit, has been diagnosed with ventilator-associated pneumonia. Culture and sensitivity testing of the client's sputum indicates that erythromycin is a treatment option. Which nursing assessment is most appropriate to rule out contraindications for this medication therapy? Question the client about any history of gastrointestinal upset. Confirm the ability to safely swallow oral medication. Review lab results to confirm normal liver function. Review the medical chart for a history of nephrotoxic signs or symptoms.

Review lab results to confirm normal liver function. Explanation: Erythromycin is seldom used in critical care settings, partly because broader-spectrum bactericidal drugs are usually needed in critically ill clients and partly because it inhibits liver metabolism and slows elimination of several other drugs. Erythromycin is not nephrotoxic. The drug is administered orally, and GI upset does not contraindicate use.

The 59-year-old client has peptic ulcer disease and is started on sucralfate. What is an appropriate nursing diagnosis related to this medication? Deficient fluid volume Risk for constipation related to GI effects Imbalanced nutrition related to nausea Risk for injury: bleeding

Risk for constipation related to GI effects Explanation: The adverse effects associated with sucralfate are primarily related to its GI effects. Constipation is the most frequently seen adverse effect. Imbalanced nutrition, if seen, would be related to diarrhea or constipation and not nausea. Fluid volume deficit and bleeding are not common adverse effects of this drug.

The nurse is caring for a client requiring digestive enzyme replacement therapy. What is the appropriate nursing diagnosis for this client? Bowel incontinence Risk for constipation Risk for imbalanced nutrition Acute pain

Risk for imbalanced nutrition The nurse would be concerned about the client's nutritional status because lack of digestive enzymes results in malnutrition. Replacement digestive enzymes help the client improve digestion and absorption of essential nutrients. Effectiveness of the therapy is determined by the client's ability to maintain balanced nutrition. The other three options are not applicable to the use of replacement digestive enzymes.

After teaching a client who is using isosorbide dinitrate for treatment of angina, the client demonstrates understanding of the information when he identifies what needs to be avoided? (Select all that apply.) Ergotamine Heparin Sildenafil Tadalafil Vardenafil

Sildenafil Tadalafil Vardenafil Sildenafil is a PDE-5 inhibitor that can cause severe hypotension and cardiovascular events when combined with nitrates. Tadalafil is a PDE-5 inhibitor that can cause severe hypotension and cardiovascular events when combined with nitrates. Vardenafil is a PDE-5 inhibitor that can cause severe hypotension and cardiovascular events when combined with nitrates. Heparin's effectiveness is decreased when combined with nitrates; dosage adjustments may be necessary. Ergot derivatives when combined with nitrates may increase the risk of hypertension and decreased antianginal effects; dosage adjustments may be necessary.

After teaching a group of students about beta-1 selective blockers, the instructor determines that the teaching was successful when the students identify what as a contraindication to using this group of drugs? Diabetes COPD Thyroid disease Sinus bradycardia

Sinus bradycardia Beta-1 selective blockers are contraindicated in patients with sinus bradycardia. Thyroid disease is a condition that requires cautious use of beta-1 selective blockers. Diabetes is a condition that requires cautious use of beta-1 selective blockers. COPD is a condition that requires cautious use of beta-1 selective blockers.

A new client presents with persistent cough, weight loss, anorexia, and night sweats. What would the nurse expect the health care provider to order? Select all that apply. Interferon gamma release assay (IGRA) Skin test Urinalysis culture and susceptibility Chest radiograph Endoscopy

Skin test Chest radiograph Interferon gamma release assay (IGRA) The described symptoms are consistent with tuberculosis, which would indicate that a chest x-ray and either a skin test or IGRA are appropriate. Urinalysis with culture and susceptibility is indicated in urinary tract infections. Endoscopy does not play a role in diagnosis of tuberculosis.

A nurse is preparing to administer a diuretic that antagonizes the action of aldosterone. Which drug would the nurse likely administer? Acetazolamide Furosemide Spironolactone Hydrochlorothiazide

Spironolactone Spironolactone is a potassium-sparing diuretic that exerts its diuretic effect by antagonizing the action of aldosterone. Furosemide is a loop diuretic when given with lithium increases the risk for lithium toxicity. Hydrochlorothiazide given with anesthetics increases the anesthetic effectiveness. Acetazolamide when given with primidone for seizures, decreases the primidone effectiveness.

The client has been prescribed amlodipine (Norvasc) for hypertension. His blood pressure upon a revisit remains elevated. The nurse inquires about the use of which alternative therapy? Prune juice Hawthorn Lavender St. John's wort

St. John's wort When taken with a calcium channel blocker, the herb St. John's wort can cause a decrease in serum levels of the calcium channel blocker. Neither hawthorn, lavender, or prune juice are known to affect serum levels of calcium channel blockers.

The nursing student studying pharmacology identifies correctly which classes of drugs to be the first to effectively treat infections? Penicillins Macrolides Sulfonamides Cephalosporins

Sulfonamides Explanation: The sulfonamides (sulfa drugs) were the first antibiotic drugs that effectively treated infections. Followed by Penicillin, Macrolides and Cephalosporins.

Clients taking levodopa should avoid what due to their ability to increase dopa decarboxylase action? Foods with a high sodium content Foods high in polyunsaturated fats Supplemental vitamins Lima beans, navy beans, and certain cereals

Supplemental vitamins Levodopa is well absorbed from the small intestine after oral administration, reaches peak serum levels within 30 to 90 minutes, and has a short serum half-life (1-3 hours). Absorption is decreased by delayed gastric emptying, hyperacidity of gastric secretions, and competition with amino acids (from digestion of protein foods) for sites of absorption in the small intestine. Pyridoxine (vitamin B6) promotes the breakdown of levodopa, reducing its effectiveness.

A nurse is working with a newly diagnosed diabetic client on understanding hypoglycemia and insulin reactions. Which action would be most important for the client to understand when planning the response to an insulin reaction? Inject a prescribed dose of insulin as soon as you suspect the reaction is occurring. Take an oral dose of some form of glucose as soon as possible. Stay calm and still until the reaction subsides. Notify your health care provider immediately.

Take an oral dose of some form of glucose as soon as possible. Explanation: The initial action of the client should be to take some form of oral glucose. It would also be appropriate to call the provider, but this will delay self-treatment and should be done after the administration of the glucose. Injecting insulin would cause further harm to the client and is not an option. It is good to stay calm, but the reaction will not subside without intervention.

A client has been receiving fluoroquinolone after being diagnosed with an infection. Which finding will the nurse anticipate noting upon evaluation? White blood count is increasing Blood pressure is returning to normal Temperature is returning to normal Changes on heart monitor

Temperature is returning to normal Explanation: One indication that the antibiotic is working is when the client's temperature returns to normal once treatment has been started. An increase in white blood cell count or temperature demonstrates the infection is worsening. Blood pressure returning to normal does not indicate a lack of infection. Changes on the heart monitor reading, specifically prolonged QT interval would be an adverse effect and the nurse would need to notify the primary care provider of this change.

The nurse administers psyllium to a client with constipation. What outcome best demonstrates therapeutic effects? The bulk of the client's stools is increased. The client has two to three bowel movements daily. The client's bowel sounds are reduced. The client remains free of diarrhea.

The bulk of the client's stools is increased. Psyllium is a natural substance that forms a gelatin-like bulk of the intestinal contents. This agent stimulates local activity. It is considered milder and less irritating than many other bulk stimulants. Clients must use caution and take it with plenty of water because psyllium absorbs large amounts of water and produces stools of gelatin-like consistency. It is not used to treat diarrhea. Two to three bowel movements daily is likely considered excessive by most clients. Reduced bowel sounds would suggest worsening of constipation, not resolution.

A hospital client has been prescribed IV ceftriaxone, a cephalosporin antibiotic. What assessment finding related to drug therapy would be of greatest concern to the nurse? The client's IV site becomes infiltrated The client's urine output is 450 mL over eight hours The client develops jaundice The client has a fever

The client develops jaundice Explanation: Jaundice can signal hepatic damage, which would necessitate discontinuing a cephalosporin. Urine output of 450 mL over eight hours is adequate. The nurse would monitor and treat the client's fever, but jaundice is a more ominous sign. Infiltration would require re-siting the client's IV, but this is not as serious as possible liver damage.

An older adult client began treatment for Parkinson disease several months ago and the nurse is assessing the client. What assessment finding should suggest that the client's medications are becoming less effective? The client's neutrophil count is gradually declining. The client has begun drooling while they eat. The client has lost interest in their favorite foods. The client is increasingly impatient with their spouse and children.

The client has begun drooling while they eat. Drooling is a characteristic sign of Parkinson disease, and a new onset of this would suggest the progression of the disease. Impatience and loss of interest would suggest a worsening mood and affect, which may or may not be related to the client's Parkinson disease. Neutropenia would likely be unrelated to the disease or its treatment.

A client with cardiac disease and taking Plavix is admitted with a lower respiratory infection. The physician starts the client on Levaquin 500 mg IV every 24 hours. About what should the nurse be concerned? The client may have an increased risk for bleeding. The client may have an increased glucose level. The client may have decreased absorption of Levaquin. The client may experience seizure activity.

The client may have an increased risk for bleeding. Explanation: When a fluoroquinolone is administered with another agent, there may be interactions. When administered with a oral anticoagulant such as Plavix, there may be an increased risk of bleeding. The other effects could be seen with other drugs, but not with Plavix.

Which condition must be met in order for glyburide treatment to be effective? The client must have hemoglobin A1C of ≤7%. The client must not have hyperglycemia. The client must be able to self-administer the medication. The client must have functioning pancreatic beta cells.

The client must have functioning pancreatic beta cells. Explanation: Because glyburide stimulates pancreatic beta cells to produce more insulin, it is effective only when functioning pancreatic beta cells are present. The presence of normal blood glucose levels would render the medication unnecessary. Self-administration is common but not absolutely necessary.

The nurse is caring for the following clients and is aware that sulfonamides should be used cautiously in which client? The client with a knee replacement The client with Bell's palsy The client with an appendectomy The client with asthma

The client with asthma Explanation: The nurse knows that sulfonamides should be used with caution in clients with renal impairment, hepatic impairment, or bronchial asthma. A client with a knee replacement, an appendectomy, or with Bell's palsy would not have to be cautious when using sulfonamides. Reference:

Which client should not receive erythromycin as ordered at 8 a.m.? The client with elevated liver enzymes The client who is 60 years old The client taking a beta blocker The client with an upper respiratory infection

The client with elevated liver enzymes Explanation: The medication is metabolized by the liver and excreted in the bile. The client with elevated liver enzymes will not be able to tolerate this medication. There is a risk it will build up in the system. The other situations are not contraindications for taking erythromycin.

The clinic nurse assesses a client taking benazepril to control hypertension. What change in the client's health status may require a change in drug therapy? The client is diagnosed with gastroesophageal reflux disease. The client is treated for hepatitis A. The client's creatinine clearance is steadily declining. The client is diagnosed with depression and begins taking a selective serotonin reuptake inhibitor (SSRI).

The client's creatinine clearance is steadily declining. Benazepril is an angiotensin-converting enzyme inhibitor; drugs in this class are contraindicated in the presence of impaired renal function. Mental illness, hepatic disease, or GERD are not contraindications with this drug.

A client has a new diagnosis of a seizure disorder. What aspect of this client's health status would contraindicate the use of carbamazepine? The client has type 2 diabetes, controlled through diet. The client's most recent blood work reveals pancytopenia. The client has an allergy to sulfonamides. The client is 17 years old.

The client's most recent blood work reveals pancytopenia. Bone marrow suppression would be considered a contraindication to administration of carbamazepine therapy. Contraindications to the use of Tegretol do not include an allergy to sulfonamides, or diabetes. The drug could be safely administered to a 17 year-old client.

A 30-year-old woman who is in the first trimester of pregnancy has presented to her primary care provider with a 4-day history of a reddened, itchy left eye that is crusted with purulent exudate. The clinician suspects a bacterial, rather than viral, etiology. How will the client's pregnancy affect the potential use of ciprofloxacin to treat her conjunctivitis? Ciprofloxacin is safe to use in pregnancy and the client may use to same dose and route as a nonpregnant client. The client will require a lower dose and longer course of ciprofloxacin than a nonpregnant, adult client. It is safe for the client to use topical ciprofloxacin but the oral route is potential teratogenic. The use of ciprofloxacin is contraindicated in pregnancy.

The use of ciprofloxacin is contraindicated in pregnancy. Explanation: Ciprofloxacin is contraindicated in clients who are pregnant or lactating. Alternative routes and dosages do not mitigate the risks during pregnancy.

A nurse is to administer an anticonvulsant drug. The nurse understands that this drug is classified as an oxazolidinedione. Which drug would the nurse administer? Levetiracetam Diazepam Trimethadione Gabapentin

Trimethadione Trimethadione is classified as an oxazolidinedione. Diazepam is classified as a benzodiazepine. Gabapentin is classified as a miscellaneous anticonvulsant. Levetiracetam is classified as a miscellaneous anticonvulsant.

In what client circumstance would the use of a bulk-forming laxative be contraindicated? inability to ambulate independently currently prescribed vitamin B supplements a history of hemorrhoids acute abdominal pain

acute abdominal pain Explanation: Laxatives and cathartics should not be used in the presence of undiagnosed abdominal pain or other signs of intestinal obstruction because of the risks of perforation and peritonitis. Vitamin supplements, hemorrhoids, and decreased mobility do not necessarily contraindicate the use of laxatives.

The nurse is working with the interdisciplinary team to plan the care of a client newly diagnosed with epilepsy. What factors should the team consider when determining the drug of choice for the client? Select all that apply. type of epilepsy age cognitive status gender culture and ethnicity

age type of epilepsy culture and ethnicity The drug of choice for any given situation depends on the type of epilepsy, client age, specific client characteristics such as cultural variations, and client tolerance for associated adverse effects as opposed to preferred adverse effect. Gender and cognition do not play a role in determining drug of choice.

A client has recently been prescribed a drug that treats hypertension by blocking the sympathetic receptors in the sympathetic nervous system. This action is characteristic of which? a cardiotonic. an adrenergic agonist. an adrenergic antagonist. a neurotransmitter.

an adrenergic antagonist. Those drugs that stimulate sympathetic receptors are referred to as adrenergic or dopaminergic agonists (stimulators), and those that block are referred to as adrenergic antagonists (blockers). Adrenergic antagonism is not synonymous with the action of a cardiotonic drug.

The action of sulfonamides is classified as which type of medication? bacteriostatic bacteriophage bacteriostationary bactericidal

bacteriostatic Explanation: Sulfonamides are primarily bacteriostatic. Drugs that destroy the bacteria are referred to as bactericidal. Bacteriostationary keeps the bacteria in place, and bacteriophage uses enzymatic substances to destroy the bacteria.

A client diagnosed with irritable bowel syndrome has been prescribed alosetron. The nurse should ensure that the client has been made aware of what potentially severe adverse effect? bowel obstruction diverticulosis anal fissures blood dyscrasias

bowel obstruction Severe constipation, with possible obstruction, perforation, and hemorrhage, is the most common problem resulting from alosetron. None of the other options are associated with this medication.

An 80-year-old client has been brought to the emergency department in shock. The client is receiving dopamine. The nurse should prioritize the assessment for what potentially serious adverse effect? renal insufficiency blood dyscrasias cardiac arrhythmia hepatic toxicity

cardiac arrhythmia Dopamine therapy can result in cardiac arrhythmias, which can be life threatening. Older clients are more likely to experience the adverse effects associated with adrenergic agonists and should be started on lower doses and monitored closely for arrhythmias and blood pressure changes. Blood dyscrasias, hepatic toxicity, and renal insufficiency are not commonly associated with dopamine use. In fact, at lower doses, dopamine increases renal perfusion.

A client has been prescribed benztropine as drug therapy for Parkinson disease. What assessment finding would suggest a therapeutic effect to the nurse? increased level of consciousness increased motivation and muscle strength absence of seizure activity decreased rigidity and tremors

decreased rigidity and tremors Benztropine should cause a decrease in rigidity and tremors. It does not affect LOC, motivation, strength, or seizure risk.

The nurse is caring for an older adult client who receives furosemide on a regular basis. The nurse should prioritize assessments related to what health problem? urinary retention dysrhythmias dehydration hypernatremia

dehydration Excessive diuresis, particularly in older adults, may cause dehydration, blood volume reduction with circulatory collapse, and the risk of vascular thrombosis and embolism. Hypernatremia, urinary retention, and dysrhythmias are unlikely to result from the use of furosemide.

A client has developed symptoms of rigidity and bradykinesia. Which medication has been linked to the development of such symptomology? furosemide valproic acid haloperidol psyllium hydrophilic mucilloid

haloperidol Drugs that deplete dopamine stores or block dopamine receptors, including the older antipsychotic drugs (phenothiazines and haloperidol), reserpine, and metoclopramide, can produce movement disorders such as secondary parkinsonism. Neither furosemide, psyllium, nor valproic acid depletes dopamine stores.

A nursing instructor is teaching about the tuberculosis (TB) drug pyrazinamide and informs students that the most severe adverse reaction to this drug is which of the following? hepatotoxicity myalgia rashes diarrhea

hepatotoxicity Some generalized reactions to pyrazinamide are nausea and vomiting, diarrhea, myalgia, and rashes. The most severe adverse reaction with pyrazinamide is hepatotoxicity.

When considering known adverse reactions to gentamicin therapy, the nurse should focus assessment of what body structure? oral cavity inner ears eyes lymph nodes

inner ears Explanation: Gentamicin reaches higher concentrations in the kidneys and inner ears than in other body tissues; this is a major factor in nephrotoxicity and ototoxicity. This situation requires focused assessment of the inner ears and kidney function. The increased concentration does occur in any of the other proposed locations.

Cephalosporins might interfere with the accuracy of which test? ketone urine test A1C test HIV test cholesterol test

ketone urine test Explanation: Clients with diabetes who use urine ketone testing for determining diabetes medication dosing and who are prescribed cephalosporins need to be aware that this drug may interfere with the accurate test results. Cephalosporins do not interfere with blood testing such as with HIV, cholesterol, or A1C.

A male client tells the nurse that he takes antacid tablets several times each day. Knowing that the brand of antacid he uses contains calcium carbonate, the nurse cautions the client that overuse could place him at risk for: hypocalcemia. metabolic acidosis. hypercholesterolemia. metabolic alkalosis.

metabolic alkalosis. Overuse of antacids containing calcium carbonate can cause alkalosis and raise urine pH.

The nurse should anticipate the administration of what drug when an older adult is diagnosed with community-acquired pneumonia? streptomycin moxifloxacin tobramycin norfloxacin

moxifloxacin Explanation: Moxifloxacin is indicated for community-acquired pneumonia. This health problem is not normally treated with streptomycin or tobramycin. Norfloxacin is used in the treatment of urinary tract infections (UTIs).

A need for additional education on the administration of epinephrine is needed when a new nursing graduate indicates that epinephrine can be effectively administered by what route? subcutaneous intravenous oral topical

oral Administration of epinephrine is by inhalation, injection, or topical application. Oral administration of the drug is not effective, because enzymes in the GI tract and liver destroy it.

An adult client with multiple chronic health problems has been prescribed furosemide in the management of hypertension. When reviewing this client's current medication administration record, what drug should signal the nurse to a potentially increased risk of hypokalemia? calcium carbonate ibuprofen vitamin D prednisone

prednisone Corticosteroids increase the risk of hypokalemia in clients who are taking furosemide. Vitamin D, calcium supplements, and nonsteroidal anti-inflammatory drugs (NSAIDs) like ibuprofen do not have this effect.

A nurse is assessing a client who is receiving vancomycin intravenously. While the drug is being administered, the client reports a throbbing sensation in their neck and back and paresthesias. The client's blood pressure has dropped and their neck and back are erythematous. The nurse suspects which adverse finding? Stevens-Johnson syndrome (SJS) pseudomembranous colitis toxic epidermal necrosis red man syndrome

red man syndrome Explanation: Red man syndrome is manifested by a drop in blood pressure and reports of throbbing neck or back pain. Additionally, the client develops fever, chills, paresthesia, and erythema (redness) of the neck and back. SJS is manifested by fever, cough, muscular aches and pains, headache, and lesions of the skin, mucous membranes, and eyes; the lesions appear as red wheals or blisters, often starting on the face, in the mouth, or on the lips, neck, and extremities. Toxic epidermal necrosis is manifested by skin lesions resulting from the death of the epidermal layer of the skin. Pseudomembranous colitis is a bacterial superinfection that is manifested by severe bloody diarrhea.

A health care provider has asked the nurse to educate a client with Parkinson disease regarding the client's medication regimen. Because the client is taking carbidopa-levodopa, the nurse will assess the client's existing medication regimen for: beta-blockers. potassium-wasting diuretics. tricyclic antidepressants. NSAIDs.

tricyclic antidepressants. Carbidopa-levodopa can interact substantially with hydantoins, MAOIs, phenothiazines, or tricyclic antidepressants. It does not interact appreciably with NSAIDs, diuretics, or beta-blockers.

A nurse is reviewing the medical record of a client who is prescribed tetracycline. Which drug, if found being used by the client, would alert the nurse to the need for a decreased dosage of that drug? losartan vancomycin atorvastatin warfarin

warfarin Explanation: The concomitant use of tetracycline with warfarin, an anticoagulant, increases the risk of bleeding, necessitating a reduction in the dosage of warfarin. Vancomycin is used in specific cases of bacterial and resistant infections. Atorvastatin is for hyperlipidemia, and losartan is an antihypertensive. Tetracycline has no drug interaction with these drugs.

A client, being evaluated for diabetes, asks how a blood glucose test is used to diagnosis this disease. What is the nurse's best response? "Two consecutive glycosylated hemoglobin (hemoglobin A1C) results of 6 or more are diagnostic of diabetes." "A fasting blood sugar result of 100 mg/dL (5.55 mmol/L) or more on two separate occasions is diagnostic of diabetes." "A fasting blood sugar result of 100 mg/dL (5.55 mmol/L) or more and an A1C of more than 6 on two separate occasions are diagnostic of diabetes." "A fasting blood sugar result of 126 mg/dL (6.99 mmol/L) or more on two separate occasions is diagnostic of diabetes."

"A fasting blood sugar result of 126 mg/dL (6.99 mmol/L) or more on two separate occasions is diagnostic of diabetes." Explanation: A major clinical manifestation of hyperglycemia is fasting blood glucose levels exceeding 126 mg/dL (6.99 mmol/L). A person with a fasting blood glucose level between 100 and 125 mg/dL (5.55 to 6.94 mmol/L) is said to have impaired fasting glucose or prediabetes. The normal hemoglobin A1C level is under 7.

A client taking ginger for motion sickness reports heartburn. What is the nurse's best response? "Ginger is a new genetically engineered herb, and all the effects are not yet known." "Ginger should be used cautiously due to the high chance of adverse reactions." "Adverse effects are rare, but heartburn has been reported by some individuals taking ginger." "The adverse effects are less likely to occur if taken for the pain and inflammation of arthritis."

"Adverse effects are rare, but heartburn has been reported by some individuals taking ginger." Explanation: Adverse reactions to ginger are rare, although heartburn has been reported by some individuals. Ginger has been consumed safely as a food for centuries. There is no data to support the opinion that taking ginger for different reasons decreases the chance for heartburn.

A nurse is teaching a client who is prescribed diphenoxylate with atropine. Which statement by the client indicates the teaching was effective? "As my stools decrease, I will discontinue this medication." "I will add this medication to my daily medication schedule." "My urine should be dark and cloudy." "I will start working out at the gym before my bowel habits become more normal."

"As my stools decrease, I will discontinue this medication." Diphenoxylate with atropine is used for diarrhea and not for daily use. Once the stools change to a more normal pattern, the medication should be stopped. Working out can lead to dehydration and fatigue. The medication does not affect urine; however, urine output that is dark and cloudy could indicate dehydration.

A male client diagnosed with bone cancer has an order for clonidine on his chart. The nurse is concerned that this may be a mistake and asks another nurse why clonidine would be ordered for this client. What would be an appropriate response from the second nurse? "Clonidine is sometimes prescribed for severe pain in cancer clients like this client." "Clonidine will not hurt him, so follow the order." "Clonidine decreases cancer cells within the bone so the provider ordered it for that reason." "Clonidine is given for hypotension, so this client must have a history of hypotension."

"Clonidine is sometimes prescribed for severe pain in cancer clients like this client." Clonidine is prescribed to treat severe pain in clients with cancer. Clonidine does not have an effect on cancer cells. A nurse should not blindly follow orders. Clonidine is given for hypertension, not hypotension.

A 4-year-old client being discharged from the hospital is ordered an oral-suspension cephalosporin. When the nurse instructs the mother to store the bottle in the refrigerator, the mother asks, "Why does that matter?" The nurse's best response would be which? "Storing this drug at room temperature will cause it to thicken and harden." "It's important to follow what the directions say." "The refrigerator is convenient and a way to track the medicine's whereabouts." "Drugs that require refrigeration lose potency if kept at room temperature."

"Drugs that require refrigeration lose potency if kept at room temperature." Explanation: Refrigeration will ensure that the drug does not lose its potency as it will if kept at room temperature. It is imperative that the nurse relays this rationale to the client's mother so she understands the importance of following the directions for storage of the medication. It will not thicken and harden, just lose potency. Mothers usually keep track of medications for their children as they do not want any harm to come to them.

A client is instructed to take diphenhydramine after an allergic reaction. Which statement by the client indicates successful teaching concerning the safe and effective use of diphenhydramine? "I can take this medication every 2 hours until I feel better." "I will eat a diet low in sodium while taking this medication." "I will still be able to have my after-dinner drink with this medication." "I should not drive my car after taking this medication."

"I should not drive my car after taking this medication." The administration of diphenhydramine (Benadryl) causes drowsiness, and the client should not operate machinery, such as driving. The client should not combine diphenhydramine with alcohol due to central nervous system depression. The client will not need to limit sodium with this medication. The client should adhere to the dosing schedule and not take the medication every 2 hours.

A nurse is working with a client who is prescribed propranolol. While assessing the client's health history, what statement should the nurse prioritize for follow-up? "Do you know how to do therapeutic touch? I've heard it can help with high blood pressure." "I've been going for acupuncture treatments twice a week to help with my high blood pressure." "I started taking Di huang supplements a few weeks ago on the advice of my naturopath." "I've read a lot about the benefits of vitamin B6 so I'm taking a supplement each morning."

"I started taking Di huang supplements a few weeks ago on the advice of my naturopath." Di huang is an alternative therapy that can lower blood glucose when used in combination with adrenergic blocking agents. This constitutes a safety risk that the nurse should address. The nurse should also ensure that the client's provider is made aware of his or her vitamin B6 use, but this likely poses less of a safety risk. Acupuncture and therapeutic touch have minimal risks and would not be a priority.

A nurse is teaching a client how to apply a topical sulfonamide to a burn. Which statement by the client would indicate that the teaching was effective? "If I feel some stinging, I should call the health care provider." "I need to fan the area while the wound is open." "I will apply it in a very thin layer over the area." "I must avoid cleaning the wound before I apply the drug."

"I will apply it in a very thin layer over the area." Explanation: When applying a topical sulfonamide, the client should apply it in a thin layer, about 1/16-in thick. Some stinging on application is normal and need not be reported. Air drafts worsen the pain and should be avoided. The wound should be cleaned and any debris removed before the drug is applied.

The nurse provides client teaching for a client diagnosed with angina about the prescribed nitroglycerin transdermal patch. Which client statement establishes the need for further teaching? "I will rotate placement of the transdermal patch on my chest, abdomen, and thighs." "I will clean the previous site where I remove an old patch with soap and water and dry it thoroughly." "I will apply the patch for 24 hours and then remove it, and place another transdermal patch." "I will remove the patch and fold it on itself to prevent my dog or grandchildren from touching it."

"I will apply the patch for 24 hours and then remove it, and place another transdermal patch." The nurse should provide further teaching about the nitroglycerin transdermal patch needs to be applied in the morning, and then removed after 10-12 hours, and left off until the next morning. The patch delivers a constant amount of the drug, which without removal of the drug leads to tolerance of the drug, making it ineffective. The client is correct in rotating sites of placement of the transdermal patch and to place it on chest, abdomen, and thighs. The client should avoid lower legs. It is important to fold the patch in half before disposing of it in the trash to prevent a person or pet from encountering the drug that remains in the delivery system. Cleaning the skin is important in removing any residual drug from the skin.

An 11 year-old client has been diagnosed with epilepsy and prescribed phenytoin 100 mg PO b.i.d. What statement by the client's parent suggests an accurate understanding of the client's medication regimen? "I will make sure my child has routine visits to the dentist." "I will stop the drug immediately if any side effects occur." "I will make sure my child takes the medication on an empty stomach." "I will weigh my child daily and feed them a high-calorie diet."

"I will make sure my child has routine visits to the dentist." Gingival hyperplasia is common in clients, especially children, who take phenytoin, which makes regular dentist visits important to oral health. Taking the medication on a full stomach or with meals reduces gastrointestinal (GI) adverse effects. The mother should call the healthcare provider if adverse effects are noted and needs to understand the risks associated with abrupt withdrawal of the medication. Daily weight taking and high-calorie diets are not necessary during phenytoin administration.

Which statement by a client taking a sulfonamide requires further instruction? "I will be sure to drink a full glass of water every time I take my medicine." "I will take my medicine with my meals like it says on the prescription bottle." "I will make sure to use extra sunscreen when I go to the tanning booth." "I will take all of my medicine even if my symptoms go away."

"I will make sure to use extra sunscreen when I go to the tanning booth." Explanation: Clients taking sulfonamides should avoid any exposure to sunlight or ultraviolet light, such as tanning beds or sunlamps. Extra sunscreen would not protect the client from the photosensitivity effect of the medication. The client is correct in stating that he would complete the entire course of the medication, would drink a full glass of water with each pill, and would take the medication with meals.

The nurse is teaching a 62-year-old client about hydrochlorothiazide, which the health care provider has prescribed for treatment of hypertension. What statement, made by the client, suggests that the client understands the teaching? "I will need to stand slowly." "I can jump right into action!" "I will need to make sure that I am consuming enough sodium." "I must take my medication on an empty stomach."

"I will need to stand slowly." Diuretics, including hydrochlorothiazide, can cause orthostatic hypotension. Rising slowly to a standing position can reduce the risk of falls. Patients taking diuretics should restrict sodium intake to avoid the need for higher doses, which increase the likelihood of adverse effects. Taking diuretics with food can help avoid the GI irritation frequently associated with these drugs.

A young man has been diagnosed with type 2 diabetes and has been prescribed glyburide. Which statement suggests that the nurse should perform further health education? "I'll keep in mind that glyburide can possibly cause me to have low blood sugar." "I'll make sure to check with my provider before I start taking any other medications." "I'll plan to take my glyburide each night before I go to bed." "I know that glyburide won't cure my diabetes, but it will help me have safe blood sugar levels."

"I'll plan to take my glyburide each night before I go to bed." Explanation: Glyburide is normally taken in the morning, before breakfast. No drug cures diabetes; the goal of therapy is the maintenance of safe blood glucose levels. The client should check before taking other drugs and should indeed be aware of the risk of hypoglycemia.

A client has been prescribed omeprazole by the primary health care provider. When the nurse asks whether the medication is providing relief, the client replies, "I think it's working quite well, and I've gotten in the routine of taking it every morning before breakfast." How should the nurse respond? "I'm glad to hear that. It sounds like you're taking it exactly like it should be taken." "That's good, but remember that you shouldn't take it on days when you're not having any symptoms." "That's great. If you find later that it's not working as well, you might want to try taking it at bedtime." "I'm glad it's working for you, but you'll probably find it works even better if you take it after eating."

"I'm glad to hear that. It sounds like you're taking it exactly like it should be taken." It is important that omeprazole be administered before food intake. Once-daily dosing is typical. The drug is not taken solely as a response to acute symptoms.

The nurse is applying silver sulfadiazine (Silvadene) to a client's burn when the client states, "Every time you put that on me it burns." What is the nurse's best response? "I will ask the health care provider if I can apply mafenide (Sulfamylon) instead." "I will contact the health care provider and ask that the medication be discontinued." "I'm sorry that it hurts, but many clients report a burning sensation when this medication is applied." "You want it to burn because then you know that it is working."

"I'm sorry that it hurts, but many clients report a burning sensation when this medication is applied." Explanation: Many clients report a burning sensation when either mafenide or silver sulfadiazine are applied. These medications are used to treat moderate to severe burns, so it would not be appropriate to discontinue the medication. A burning sensation is not an indicator of healing tissue.

The nurse is providing education to an older adult client who has been prescribed tetracycline. Which statement should the nurse include in the teaching? "Brush your teeth twice a day to prevent discoloration." "Temporarily withhold your other medications while taking tetracycline." "Return in 5 days for a urine test to check kidney function." "Increase your fluids to 2 L or more per day to prevent renal crystals."

"Increase your fluids to 2 L or more per day to prevent renal crystals." Explanation: A major concern with the use of tetracyclines and sulfonamides in older adults is renal impairment, which commonly occurs in this population. As with younger adults, a fluid intake of 2 L daily is needed to reduce formation of crystals and stones in the urinary tract. Testing for kidney function is not routinely scheduled. Tooth discoloration is not a concern for an older adult. There is not a need to withhold other medications unless specifically instructed to do so by a member of the health care team.

The nurse is evaluating a client's understanding of the adverse reactions related to the use of theophylline. Which of the following statements by the client demonstrates an understanding of theophylline? "This medication may lower my blood pressure." "My sleep pattern should improve while I am taking this medication." "This medication may be used to treat peptic ulcers, too." "It common to experience palpitations and flushing."

"It common to experience palpitations and flushing." Adverse reactions to theophylline include restlessness, irritation, headache, nervousness, tremors, tachycardia, palpitations, and flushing. It may increase the blood pressure and is contraindicated in clients with peptic ulcer disease.

The male client is prescribed prazosin (Minipress) for hypertension. He asks the nurse how the medication works. What is the nurse's best response? "It works by inhibiting the movement of calcium across the membrane." "It promotes the excretion of sodium and water." "It works by making your blood vessels dilate." "It increases the supply of oxygen to your heart."

"It works by making your blood vessels dilate." Prazosin (Minipress) is a peripherally acting antiadrenergic drug that causes the blood vessels to expand. Calcium channel blockers inhibit the movement of calcium across the membrane and increase the supply of oxygen to the heart.

When teaching the client to safely administer nitroglycerin ointment, the nurse should convey which instruction? "Don't apply the ointment unless you're experiencing chest pain at the time." "Massage the ointment into your skin for 10 to 15 seconds after applying it." "The backs of your hands and the tops of your feet are ideal sites for applying the ointment." "Make sure you squeeze the ointment on to a paper measuring scale before applying it." SUBMIT ANSWER

"Make sure you squeeze the ointment on to a paper measuring scale before applying it." Application of nitroglycerin ointment requires using the dose-measuring application papers supplied with ointment. It is necessary to squeeze the ointment onto a measuring scale printed on paper. The ointment should be applied onto a nonhairy area of the skin, and distal extremities should be avoided. The area of application should not be massaged. All these practices would affect the absorption of the medication.

A client was given a new prescription for a transdermal nitroglycerin patch. What client education should the nurse provide regarding this medication? "Remove your old patch and put on a new one each morning." "Apply one additional patch if you experience a sudden onset of chest pain." "If your patch loses its adhesion during the day, replace it with a new one." "Put your patch on a part of your skin that doesn't have too much hair on it."

"Put your patch on a part of your skin that doesn't have too much hair on it." Transdermal nitroglycerin patches should be applied to a hairless, or nearly hairless, area of the skin to promote absorption. Loose patches should be reinforced rather than replaced. Patches should be discontinued in the evening and they are not used for acute treatment of chest pain.

A client diagnosed with Parkinson's disease asks if there are any holistic herbs or exercises that may help her balance. Which of the following would be the nurse's best response, based on previous clients' success using this approach? "Frequent dancing to music with a quick rhythm has been shown to help improve balance in clients with Parkinson's disease." "Tai chi has been shown to help clients diagnosed with Parkinson's disease improve their balance." "There are many herbs that help Parkinson's disease." "Aerobic exercises will help keep your cardiovascular system healthy."

"Tai chi has been shown to help clients diagnosed with Parkinson's disease improve their balance." Tai chi has been shown to improve balance in clients diagnosed with Parkinson's disease. No herbs are known to increase balance in clients with Parkinson's disease. Neither aerobic exercises nor dancing have been shown to improve balance in clients with Parkinson's disease.

A client has been diagnosed with a sinus infection, and the client has been given a prescription for amoxicillin. What teaching point should the nurse make for the client? "Take your medication every 8 hours, as it's been prescribed." "Make sure you tell your prescriber if you're feeling particularly tired." "This might cause crystals in your urine, so drink plenty of fluids." "Avoid taking the medication right before bed so it doesn't cause frequent trips to the bathroom."

"Take your medication every 8 hours, as it's been prescribed." Explanation: Amoxicillin is almost always given q8h. This necessitates a bedtime dose for most clients, and the drug has no diuretic effect. Extreme or longstanding fatigue should be reported, but this is unlikely to be an adverse effect of the medication.

A client has been prescribed tetracycline for the treatment of acne. How would the nurse explain the action of the drug? "Tetracycline interferes with components of bacterial protein synthesis." "Tetracycline interferes with the growth of the bacteria causing the pimples." "Tetracycline changes the chemical structure of your skin so it's less hospitable to bacteria." "Tetracycline reduces the number of bacteria in the blood running through your skin."

"Tetracycline interferes with the growth of the bacteria causing the pimples." Explanation: Tetracycline interferes with the production of free fatty acids and suppresses the growth of bacteria in sebum. These actions decrease the inflammatory, pustular lesions associated with severe acne. None of the other options accurately describe the action of tetracycline.

A client has hypertension and wants to take a nasal decongestant for cold symptoms. Which statement by the nurse indicates the effect of nasal decongestant medications on hypertension? "The administration of a nasal decongestant will increase blood pressure due to vasoconstriction of blood vessels." "The administration of a nasal decongestant will cause bradycardia and increase peripheral blood pressure." "The administration of nasal decongestant will act on the central nervous system to cause vasodilation of blood vessels." "The administration of a nasal decongestant will decrease thyroid activity, thereby increasing blood pressure."

"The administration of a nasal decongestant will increase blood pressure due to vasoconstriction of blood vessels." "The administration of nasal decongestants will increase blood pressure due to vasoconstriction of blood vessels" is indicative of good client teaching. Nasal decongestants do not cause bradycardia to increase blood pressure. The administration of nasal decongestants does not increase blood pressure through alteration of thyroid function. The administration of a nasal decongestant does not act on the central nervous system.

A client scheduled for a bowel resection is to receive neomycin sulfate by mouth. The client asks the nurse the purpose of this medication. What is the most appropriate response the nurse can provide to the client? "The administration by mouth will prevent renal damage from occurring." "The administration will decrease the growth of intestinal bacteria." "The administration by mouth will minimize the risk of ototoxicity." "The administration decreases the risk of airborne contamination of the wound."

"The administration will decrease the growth of intestinal bacteria." Explanation: Neomycin can be given before bowel surgery to suppress intestinal bacterial growth. Neomycin when administered orally is poorly absorbed in the GI tract and so exerts local bactericide effect there. The administration of neomycin will not prevent renal damage or ototoxicity but can actually increase the risk. It will also not affect the risk of airborne contamination since is focused on local effects in the GI tract.

The nurse has provided health teaching for a 15-year-old client newly diagnosed with asthma. What statement made by the client indicates a good understanding of the teaching the nurse has done regarding inhalers? "I should insert the inhaler about 1 inch into my mouth." "The aerosol canister should be shaken well before using." "I should hold my breath when administering a puff." "I need to take 3 short, quick breaths when I administer the inhaler."

"The aerosol canister should be shaken well before using." Inhalers should be shaken well, immediately before each use. It would not be appropriate to teach the client to hold his breath when administering a puff because this would inhibit inhalation. The client should hold the device around one inch from the open mouth, not inside it. There is no need to take three quick breaths.

The nurse is preparing to discharge a client who has been prescribed telithromycin. The client understands the unique adverse effect associated with this drug when the client makes which statement? "I know that this drug can make me constipated so I will take a laxative every evening." "This drug may cause visual disturbances so I won't drive until I am finished with the medication." "It is common to have a rash when taking this medication, so I will use an anti-itch cream." "Some people develop fevers while taking this drug, so I will take aspirin every four hours."

"This drug may cause visual disturbances so I won't drive until I am finished with the medication." Explanation: Visual disturbances may occur with telithromycin, so the client must be aware of this. Telithromycin causes diarrhea, not constipation, and neither rash nor fever are adverse effects of this drug.

A postoperative surgical client, prescribed twice-daily administration of docusate sodium, is concerned about developing diarrhea. What should the nurse teach the client about docusate sodium? "You'll usually have a bowel movement within 2 to 3 hours of taking this." "This drug will help you pass regular stools while you're in the hospital, but you should not take it for more than 5 days." "This medication will cause your bowels to contract more strongly than they normally do." "This medication will only soften your stools over the next couple of days."

"This medication will only soften your stools over the next couple of days." Surfactant laxatives (e.g., docusate calcium or docusate sodium) decrease the surface tension of the fecal mass to allow water to penetrate into the stool. They also act as a detergent to facilitate admixing of fat and water in the stool. As a result, stools are softer and easier to expel. These agents have little, if any, laxative effect. Their main value is to prevent straining while expelling stool. They usually act within 1 to 3 days and should be taken daily.

When evaluating an asthmatic client's knowledge of self-care, the nurse recognizes that additional instructions are needed when the client makes which of the following statements? "When I can do some, but not all of my usual activities, I am in the yellow zone." "I will wash my sheets weekly." "When I am short of breath, I will increase the use of my fluticasone." "I need to inhale my medication and hold my breath for 10 seconds."

"When I am short of breath, I will increase the use of my fluticasone." Fluticasone is an inhaled steroid and should not be used during acute asthmatic attacks. Washing linens weekly will decrease the incidence of dust mites. When clients can participate in some, but not all of usual activities, they are in the yellow zone. Once medication is inhaled, the client should hold his or her breath for 10 seconds so that medication can reach deep into the lungs.

What is the nurse's priority assessment question before giving a female client her prescription for an angiotensin II-receptor blocker (ARB)? "Have you always weighed 130 pounds as an adult?" "How much physical exercise do you get?" "When was your last menstrual period (LMP)?" "Do you eat something when you take your medications?"

"When was your last menstrual period (LMP)?" It would be important to know when the client's LMP occurred and that the client was not pregnant. These drugs can cause fetal abnormalities and fetal death. The other questions are appropriate and would help the nurse plan care for the client; however, it would not be as important as assessing for the possibility of pregnancy before beginning therapy. The nurse should teach the client the need to avoid pregnancy using a barrier contraceptive.

The nurse is providing client teaching with a client who is newly diagnosed with epilepsy. The client asks, "Can I still drive to work?" What is the nurse's best response? "You likely won't be able to until your seizures are controlled by medication" "Yes, as long as your health care provider agrees and you take your medications regularly." "You can drive as soon as therapeutic drug levels are established." "You'll need to use public transportation because a seizure could occur anytime."

"You likely won't be able to until your seizures are controlled by medication" Clients newly diagnosed with epilepsy will not be able to drive. However, after the client's seizures are controlled (usually for 6 months to 2 years depending on state law), the client may be able to regain the ability to drive. This does not, however, mean that the client has to use public transportation. Serum drug levels are not the determining criterion.

The client has been taking a fluoroquinolone and now reports that he has a white patch in his mouth. What is the best response of the nurse? "That is the medication working. Continue to take as directed." "You may be experiencing an additional infection. I will discuss this with your health care provider." "You are experiencing an adverse affect of the medication. Those symptoms will decrease over time." "You are allergic to the medication. Stop taking it immediately."

"You may be experiencing an additional infection. I will discuss this with your health care provider." Explanation: A burning sensation of the mouth or throat may be an indication of a superinfection. It is important the nurse notify the provider. The nurse should not minimize the client's concerns or cause panic.

A nurse instructing a client about use of an antitussive prescription at home should also include information about increasing fluids (unless contraindicated). How much fluid should the nurse tell this client to drink each day? 2000-3000 mL 1000-1500 mL 1500-2000 mL 500-1000 mL

1500-2000 mL When a client is prescribed an antitussive, the nurse should tell the client to drink plenty of fluids (if not contraindicated by disease process). A fluid intake of 1500 to 2000 mL/day is recommended.

The nurse should counsel a client to discontinue use of over-the-counter antidiarrheals and seek treatment from a health care provider if diarrhea persists for how long? 2 days 1 day 12 hours 7 days

2 days The nurse should counsel a client to discontinue use of over-the-counter antidiarrheals and seek treatment from a health care provider if diarrhea persists for more than 2 days. Diarrhea lasting more than 2 days can indicate infection or a condition that will require more intense treatment with prescription medication. Dehydration can occur if untreated. Diarrhea of 12 hours to 1 day can be viral. Diarrhea for 7 days can be life threatening.

A client asks the nurse about using dextromethorphan to relieve a cough. What type of cough would the nurse explain is best treated with the drug? An occasional, productive cough A dry, nonproductive cough A cough that occurs when the client is exposed to airborne irritants A cough that is associated with an allergy to ragweed

A dry, nonproductive cough The major clinical indication for use of dextromethorphan is a dry, hacking, nonproductive cough that interferes with rest and sleep. It is not desirable to suppress a productive cough because the secretions need to be removed. The character, not cause, of the cough is relevant to its treatment with dextromethorphan.

A client with ulcers has asked the nurse if it would be acceptable to take bismuth subsalicylate. Before recommending an over-the-counter (OTC) formulation of bismuth subsalicylate, the nurse should conduct which client assessment? Immunization history Cognition Allergy status Normal bowel pattern

Allergy status Because it is a salicylate, this drug can cause serious bleeding problems when used alone in clients with ulcers. People with an allergy to aspirin or other salicylates should not take bismuth subsalicylate. The client's bowel pattern, immunization history, and cognition are not central considerations.

The nurse is caring for four clients. Which client would benefit most from misoprostol, a synthetic prostaglandin E analog? A 12-year-old male with obsessive-compulsive disorder A 22-year-old pregnant female A 46-year-old female with hypertension An 83-year-old male with rheumatoid arthritis

An 83-year-old male with rheumatoid arthritis Misoprostol is prescribed for concurrent use with NSAIDs to prevent NSAID-induced erosion and ulceration of the gastric mucosa. The client with rheumatoid arthritis likely uses NSAIDs frequently, placing him at risk of developing gastric ulcers and making him a candidate for misoprostol.

The nurse calculates the infusion rate for administering dopamine to a premature infant in the neonatal intensive care unit who is in cardiogenic shock secondary to a cardiac anomaly. What is the nurse's next priority action? Obtain permission from parents. Ask another nurse to perform independent calculation. Insert an intravenous catheter. Show the nurse's calculations to the healthcare provider.

Ask another nurse to perform independent calculation. It is good practice to have a second person check the dosage calculation before administering the drug to avoid potential toxic effects. When having calculations double checked, it is best to let the other person work out separate calculations rather than just looking at the nurse's calculation first because this will be more likely to catch an error. The nurse would have another nurse perform calculations rather than the doctor. Only after calculations are correct would the drug be administered, usually through a central line or the nurse may establish a peripheral line. Permission from parents is not required above general permission needed to care for the neonate.

A client goes to his primary care provider for a cut finger. The provider is going to put sutures into the tip of the client's finger. Lidocaine with 2% epinephrine is ordered verbally by the provider. What should the nurse do? Hand him the vial of lidocaine with 2% epinephrine. Hand him a vial of lidocaine without epinephrine and say, "This is all we have today". Ask if he really wants epinephrine in the lidocaine since he is working on the tip of the finger. The nurse remembers from her reading that epinephrine is contraindicated in fingertips. Draw up the lidocaine with epinephrine and give him the syringe.

Ask if he really wants epinephrine in the lidocaine since he is working on the tip of the finger. The nurse remembers from her reading that epinephrine is contraindicated in fingertips. The use of epinephrine in lidocaine is common when placing sutures. However, epinephrine vasoconstricts the blood vessels and is not to be used on the tips of fingers, noses, or ears because necrosis of the tissue is possible. If a nurse knows something is wrong, he or she should speak up to prevent errors from occurring. By lying and saying that lidocaine without epinephrine is the only thing available, it may keep this client's finger safe, but perhaps not the next finger that the same provider sutures.

A client receives a dose of insulin lispro at 8 AM. The nurse would be alert for signs and symptoms of hypoglycemia at which time? Between 12 noon and 8 PM Between 8:30 AM and 9:30 AM Between 2 PM and 4 PM Between 10 AM and 12 noon

Between 8:30 AM and 9:30 AM Explanation: With insulin lispro, peak effects would occur in 30 to 90 minutes or between 8:30 AM and 9:30 AM. Regular insulin peaks in 2 to 4 hours, so the nurse would be alert for signs and symptoms of hypoglycemia at this time, which would be between 10 AM and 12 noon. With insulin detemir, peak effects would occur in 6 to 8 hours, or between 2 PM and 4 PM. With NPH insulin, peak effects would occur in 4 to 12 hours, or between 12 noon and 8 PM.

t is determined that a client in a hepatic coma needs a laxative. Lactulose is prescribed. What should the nurse monitor to assess the efficacy of the lactulose therapy? Blood ammonia levels Water levels in the colon Relief from symptoms Oncotic pressure in the colon

Blood ammonia levels Similar to the saline laxative magnesium hydroxide, lactulose pulls water into the colon. Unlike magnesium hydroxide, lactulose is a synthetic disaccharide of lactose. In the colon, bacteria metabolize lactulose into acids and carbon dioxide, increasing the oncotic pressure in the colon and drawing water into the stool. The acids formed also draw ammonia into the stool. Therefore, lactulose is used not only to treat constipation but also to decrease blood ammonia levels in clients in hepatic coma and those with hepatic encephalopathy. The other factors, although significant, would not indicate the efficacy of the lactulose therapy as accurately as blood ammonia levels.

A critical care nurse is preparing to administer an intragastric drip of an antacid to a client through a nasogastric tube. How should the nurse most accurately titrate the dose and frequency? By assessing the pH of a 24-hour urine sample By aspirating stomach contents and measuring the pH By swabbing the client's buccal mucosa and testing for pH daily By measuring the pH of urine after each void

By aspirating stomach contents and measuring the pH For clients with a nasogastric tube in place, antacid dosage may be titrated by aspirating stomach contents, determining pH, and then basing the dose on the pH. Accurate measurement of gastric pH cannot be determined from the buccal mucosa or urine.

A client previously experienced an anaphylactic reaction to penicillin G. Which medication should not be administered to this client due to the potential for cross-sensitivity? Ketoconazole Lactulose Kanamycin Cefadroxil

Cefadroxil Explanation: Cefadroxil is a cephalosporin. Administration of cephalosporins or carbapenems should be avoided if possible in people with life-threatening allergic reactions to penicillin. Lactulose reduces blood ammonia by resident intestinal bacteria. It is not contraindicated in the event of penicillin anaphylaxis. Ketoconazole is an antifungal and does not possess cross-sensitivity to penicillin. Kanamycin is an aminoglycoside and does not possess cross-sensitivity to penicillin.

The nurse is preparing to administer a mixture of 12 units regular insulin and 45 units NPH insulin to a client with a blood sugar of 378 mg/dL. After the nurse draws the medication into the syringe, what is the nurse's next action? Administer the insulin to the client. Check the dosage with another nurse. Check the client's blood sugar again. Ensure a meal tray is available.

Check the dosage with another nurse. Explanation: After preparing the syringe with insulin, the nurse should then have the medication and dosage checked by a second nurse to make sure that it is correct. It is not necessary to recheck the client's blood sugar again. It is important to know when the client will be eating again; make sure that it is within the next 30 minutes. However, this is not the nurse's next step. Then the nurse will administer the insulin to the client.

A client is given theophylline to treat acute asthma symptoms. Which food should the client avoid? Bananas Orange juice Chocolate Cranberry juice

Chocolate Chocolate contains caffeine and is also a xanthine; thus chocolate should be avoided when the client is taking theophylline. Restriction of bananas, orange juice, and cranberry juice is not required.

A client in the clinic with a history of epilepsy has just tested positive for pregnancy. What is the nurse's next action? Consult with the primary health care provider. Explain how to taper off her seizure medication as the risk for seizures decreases during pregnancy. Advise the client to continue her current medication for epilepsy. Discuss the most serious birth defects that may occur due to treating epilepsy during pregnancy.

Consult with the primary health care provider. The nurse should first consult with the primary health care provider. Research suggests an association between (a) the use of anticonvulsants by pregnant women with epilepsy and (b) an increased incidence of birth defects. The use of anticonvulsants is not discontinued in pregnant women with a history of major seizures because of the danger of precipitating status epilepticus. However, when seizure activity poses no serious threat to the pregnant woman, the primary health care provider may consider discontinuing use of the drug during pregnancy.

The client has been taking levofloxacin IV since admission 12 hours ago for a urinary tract infection. The nurse assesses the client's temperature at 99.8ºF. What is the nurse's best response? Administer an extra dose of levofloxacin. Administer an antipyretic. Notify the health care provider. Continue to monitor vital signs.

Continue to monitor vital signs. Explanation: The provider should be notified if the client's temperature is greater than 101ºF. The nurse cannot discontinue or administer additional doses without a provider's order. The body's normal defense to infection is an elevated temp until it reaches 101 degrees. Only then would an antipyretic be given if ordered.

Diphenoxylate with atropine sulfate resolves diarrhea by what mechanism of action? Inhibiting cellular division in causative bacteria Decreasing intestinal motility Absorbing toxins Binding with fecal material to increase bulk

Decreasing intestinal motility Explanation: Diphenoxylate with atropine sulfate resolves diarrhea by decreasing intestinal motility.

A client has been noncompliant with the diabetic medication regimen and develops diabetic ketoacidosis. Which would the nurse assess? Edema Decreased blood glucose levels Deep respirations Sour breath odor

Deep respirations Explanation: Deep respirations are seen with diabetic ketoacidosis as the body attempts to rid itself of high acid levels. The client with diabetic ketoacidosis is typically dehydrated. Blood glucose levels are typically elevated with diabetic ketoacidosis. A fruity breath odor is indicative of diabetic ketoacidosis.

The client is started on a transdermal nitroglycerin patch system. The nurse teaches the client that the system relieves pain by which action? Dilates arteries and veins Decreases the size of the artery Dilates the coronary arteries Increases the workload of the heart

Dilates arteries and veins Nitrates act by relaxing the smooth muscle of the blood vessels (arteries and veins) and increasing the amount of blood through the vessel. They also decrease the workload of the heart.

A client with type 1 diabetes has been admitted to the hospital for orthopedic surgery and the care team anticipates some disruptions to the client's blood glucose levels in the days following surgery. Which insulin regimen is most likely to achieve adequate glycemic control? Small doses of long-acting insulin administered four to five times daily Doses of basal insulin twice daily with regular insulin before each meal Large doses of rapid-acting insulin combined with long-acting insulin each morning and evening Divided doses of intermediate-acting insulin every 2 hours, around the clock

Doses of basal insulin twice daily with regular insulin before each meal Explanation: Subcutaneous insulin therapy for type 1 diabetes frequently consists of daily injections of mixtures of short-acting regular insulin with intermediate-acting insulins; multiple doses of regular insulin before each meal in association with one or two daily doses of long-acting insulin may also be used. Frequent, fixed doses of rapid-acting or intermediate-acting insulin may result in unsafe blood sugar levels.

A client being treated for cellulitis with a cephalosporin asks what the essential difference is between generations of this medication. The nurse should respond to the client's question based on what fact? The generations represent the order in which the drugs should be utilized clinically. The generations of cephalosporins represent formulations that produce fewer side effects. Each successive generation is more effective against gram-negative microorganisms. Each generation of cephalosporins has a different mechanism of action.

Each successive generation is more effective against gram-negative microorganisms. Explanation: Cephalosporins are grouped into generations by their antimicrobial properties. Each newer generation of cephalosporins has significantly greater gram-negative antimicrobial properties than the preceding generation, with decreased activity against gram-positive organisms. None of the remaining options accurately describe the concept of generations associated with medications.

Alpha- and beta-specific adrenergic agonists are primarily used to treat obstructive pulmonary conditions. True False

False Beta-specific agonists are used primarily to treat obstructive pulmonary conditions; alpha- and beta-specific agonists are used to treat hypotensive states.

The nurse is preparing to administer phenytoin (Dilantin) to a client. The nurse should notify the healthcare provider before administration based on which assessment finding? Hepatic function tests within normal parameters. Heart rate 50 beats per minute on cardiac monitor. Negative qualitative HCG test result. Blood pressure reading of 130/80 mmHg.

Heart rate 50 beats per minute on cardiac monitor. Phenytoin is contraindicated in clients with sinus bradycardia. A negative qualitative HCG test result indicates a negative pregnancy test result. Phenytoin is contraindicated in pregnancy and lactation, and is classified as a pregnancy category D drug. A slightly-elevated systolic blood pressure is not a contraindication for the administration of phenytoin. Anticonvulsants should be used cautiously in clients with kidney disease or liver disease. Normal hepatic function test results do not indicate liver disease.

A male client with diabetes who controls his disease with an oral agent and diet has been hospitalized with a urinary tract infection. The physician has ordered sulfasalazine 500 mg QID. What should the nurse closely monitor for in this client? Hypoglycemic reaction Hyperglycemic reaction Decreased potassium level Increased potassium level

Hypoglycemic reaction Explanation: When clients with diabetes are prescribed sulfonamides, the nurse should assess for a possible hypoglycemic reaction. Sulfonamides may inhibit the hepatic metabolism of the oral hypoglycemic drugs (Orinase and Diabinese). The potassium level is not affected when a client is taking a sulfonamide.

A health care prescriber recently ordered a change in medication for a patient with a seizure disorder. Which action would the nurse take in evaluating the new drug's control of the seizure disorder? Interview and observe for adverse effects Evaluate vital signs Interview and observe for evidence of underdosing or overdosing Check laboratory reports for evidence of hypernatremia

Interview and observe for adverse effects Evaluation of the drug's effectiveness includes interviewing and observing for decrease in or absence of seizure activity; interviewing and observing for avoidance of adverse drug effects, especially those that impair safety; and, when available, checking laboratory reports of serum drug levels for therapeutic ranges or evidence of underdosing or overdosing.

A nurse is completing an assessment on a client experiencing diarrhea. The client states having diarrhea for the past few days, but prior to the diarrhea experienced one week of constipation. Based on the description, what does the nurse suspect? Peripheral vascular disease Irritable bowel syndrome Ulcerative colitis Laxative abuse

Irritable bowel syndrome Explanation: Irritable bowel syndrome is a functional disorder of intestinal motility with no evidence of inflammation or tissue changes. It can include a change in bowel pattern (constipation, diarrhea, or a combination of both) accompanied by abdominal pain, bloating, and distention or presenting symptoms. Ulcerative colitis is diarrhea with mucus, proteins, and blood. Peripheral vascular disease is pain in the lower extremities. Laxative abuse is characterized by use of laxative, which is not stated by the client.

A nurse is completing an assessment on a client experiencing diarrhea. The client states having had diarrhea for the past few days but prior to the diarrhea experienced one week of constipation. Based on the description, what does the nurse suspect? Laxative abuse Ulcerative colitis Peripheral vascular disease Irritable bowel syndrome

Irritable bowel syndrome Irritable bowel syndrome is a functional disorder of intestinal motility with no evidence of inflammation or tissue changes. It includes a change in bowel pattern (constipation, diarrhea, or a combination of both) accompanied by abdominal pain, bloating, and distention or presenting symptoms. Ulcerative colitis is diarrhea with mucus, proteins, and blood. Peripheral vascular disease is pain in the lower extremities. Laxative abuse is characterized by excessive use of laxatives.

A client has been prescribed acarbose. What is the advantage of acarbose over alternative drugs? It can replace the use of insulin. The client does not have to limit food intake. It prevents alkalosis. It delays the digestion of complex carbohydrates.

It delays the digestion of complex carbohydrates. Explanation: Acarbose delays the digestion of complex carbohydrates into glucose and other simple sugars. Acarbose may be combined with insulin or an oral agent, usually a sulfonylurea. The client will still need to remain on a diabetic dietary regime. The drug does not directly prevent acid-base imbalances.

The nurse is educating a client who has just been diagnosed with tuberculosis (TB). The nurse informs the client that the drug regimen is extensive. What should the nurse tell this client about the drug therapy related to duration? It usually lasts 3 to 6 months. It usually lasts 2 months. It usually lasts 6 to 9 months. It usually lasts 1 to 2 years.

It usually lasts 6 to 9 months. Treatment for TB is divided into two phases: the initial phase and continuing phase. The initial phase usually lasts 2 months, and the continuing phase lasts approximately 4 months, with the total treatment period lasting 6 to 9 months.

A client, diagnosed with alcoholism and chronic liver failure, is confused and has an elevated serum ammonia level. What laxative will be administered to lower the serum ammonia level? Docusate sodium Polyethylene glycol-electrolyte solution Lactulose Sorbitol

Lactulose Lactulose decreases production of ammonia in the intestine. Lactulose is a disaccharide that is not absorbed from the GI tract. It is used to treat hepatic encephalopathy. Docusate sodium only provides stool softening and has no effect on serum ammonia levels. Polyethylene glycol-electrolyte solution is not used to decrease serum ammonia. Sorbitol is not used to decrease serum ammonia.

A 40-year-old is being treated for an ear infection with a cephalosporin. Which adverse reactions should the nurse monitor for in the client? Nausea Excessive tearing Chest pain Hypotension

Nausea Explanation: The most common adverse reactions that are caused due to cephalosporin administration include nausea, vomiting, and diarrhea. Cephalosporin does not cause hypotension, chest pain, or excessive tearing; hypotension and chest pain are some of the adverse reactions of disulfiram.

The nurse recognizes that which medication may be used to treat a hypertensive crisis? Amlodipine Minoxidil Hydralazine Nitroprusside

Nitroprusside Nitroprusside is administered during a hypertensive crisis intravenously. Hydralazine and minoxidil are used for severe hypertension, and amlodipine is also administered for hypertension.

The client is being started on oral ciprofloxacin (Cipro). The nurse identifies that the client is also taking corticosteroids. What is the most appropriate action of the nurse? Notify the health care provider Monitor for adverse reactions Teach about adverse reactions Decrease the dosage of Cipro

Notify the health care provider Explanation: Clients taking fluoroquinolones and corticosteroids are at risk for tendonitis and tendon rupture. The nurse needs to make sure the provider is aware that the client is taking corticosteroids. It is outside the scope of nursing to adjust medication dosages. Adverse reactions can be discussed after the provider determines that the medication is safe for the client.

A male client is having angina. As the nurse prepares to administer sublingual nitroglycerin, it is noted that the client's blood pressure is 82/54. What is the best action of the nurse? Ask the client how he feels. Withhold medication until blood pressure increases. Notify the health care provider. Administer the medication.

Notify the health care provider. The client is having angina and it must be addressed. If the blood pressure is less than 90 systolic, the nurse should not administer the medication and call the provider immediately.

A client is hospitalized due to nonadherence to an antitubercular drug treatment for a diagnosis of tuberculosis (TB). Which intervention is most important for the nurse to implement? Administering the medications parenterally Instructing the family on the medication regime Counting the number of tablets in the bottle daily Observing the client taking the medications

Observing the client taking the medications Directly observed therapy in which a health care provider observes the client taking each dose of anti-TB drugs is recommended for all drug regimens and is considered mandatory in this case. The medications are not administered parenterally. The family should be instructed on the medication regime, but this action is not imperative in maintaining compliance. Tablets missing from the bottle may not necessarily have been taken correctly by the client.

A 36-year-old client is prescribed inhaled corticosteroid (ICS) for daily use. Which adverse effects should the nurse closely monitor for in this client? Delayed growth Oropharyngeal Candida albicans infection Hypotension Suppression of the hypothalamic-pituitary-adrenal axis

Oropharyngeal Candida albicans infection Dysphonia and oropharyngeal Candida albicans infections are common adverse effects associated with daily use of ICS. Long-term use of ICS may delay growth in children, not adults. Suppression of the hypothalamic-pituitary-adrenal axis is a rare adverse effect, and hypotension is not an identified adverse effect of the drug.

Which is classified as a decongestant? Azelastine (Astelin) Oxymetazoline (Afrin) Levocetirizine (Xyzal) Dextromethorphan (Delsym)

Oxymetazoline (Afrin) Afrin is a decongestant. Xyzal is a second-generation antihistamine. Delsym is a centrally acting antitussive that contains dextromethorphan. Astelin is a second-generation antihistamine.

Antacids have not been effective in managing a client's gastroesophageal reflux disease, so the health care provider is prescribing a proton pump inhibitor. Why might the health care provider prescribe a proton pump inhibitor (PPI) rather than a histamine2 receptor antagonist (H2RA)? PPIs suppress acid more strongly and for a shorter time. PPIs are less expensive. PPIs suppress acid more strongly and for a longer time. H2RAs do not suppress acid.

PPIs suppress acid more strongly and for a longer time. Explanation: Compared with H2RAs, PPIs suppress gastric acid more strongly and for a longer time. This effect provides faster symptom relief and faster healing in acid-related diseases.

A family brings a client to the emergency department because the client has been hallucinating and, when in a deep sleep, stops breathing when not stimulated. The nurse learns the client has been self-treating diarrhea and suspects the client was taking what medication? Bismuth subsalicylate Colace Paregoric Loperamide

Paregoric Opium derivatives, like paregoric, are associated with light-headedness, sedation, euphoria, hallucinations, and respiratory depression related to their effect on opioid receptors. Nonopioids such as bismuth subsalicylate and loperamide would not cause respiratory depression. Colace is a stool softener, not an antidiarrheal.

After teaching a group of nursing students about seizures, the instructor determines that the teaching was successful when the group identifies seizures that do not impair consciousness but can involve the senses or motor ability as which type? Myoclonic seizures Generalized seizures Tonic-clonic seizures Partial seizures

Partial seizures Seizures that do not impair consciousness but can involve the senses or motor ability are classified as partial seizures. Generalized seizures involve loss of consciousness during the seizure. Tonic-clonic seizures are a type of generalized seizure involving alternate contraction and relaxation of the muscles, loss of consciousness, and abnormal behavior. Myoclonic seizures involve sudden, forceful contractions of single or multiple groups of muscles.

Michael, 25 years old, has had mitral valve regurgitation since age four, after having rheumatic fever. Michael is planning to go to his dentist to have his teeth cleaned. Because of Michael's history he will need to take antibiotics in conjunction with this procedure to prevent bacteremia. Which class of antibiotics will Michael most likely receive if he has no allergies? Penicillin Tetracycline Vancomycin Cephalosporin

Penicillin Explanation: Penicillin G may also be used as prophylaxis in special patient populations to prevent bacterial endocarditis prior to procedures likely to produce temporary bacteremia, such as dental procedures. These patients include those with prosthetic heart valves, mitral valve prolapse, most congenital heart diseases, and acquired valvular heart disease. It may also be used as prophylaxis in patients with recurrent rheumatic fever or rheumatic heart disease.

A female client is diagnosed with type 1 diabetes. She suddenly reports feeling weak, shaky, and dizzy. What should the nurse's first response be? Administer 10 units of regular insulin subcutaneously. Administer 1 amp of 50% dextrose IV. Perform a blood sugar analysis. Have the client drink a glass of orange juice.

Perform a blood sugar analysis. Explanation: As long as the client is awake and verbally responsive, check the blood glucose level first. Hypoglycemia can make the client feel weak, confused, irritable, hungry, or tired. Clients may also report sweating or headaches. If the client has any of these symptoms, check the blood glucose. If the level is 70 mg/dL or below, have the client consume 3 or 4 glucose tablets; 1 serving of glucose gel; 1/2 cup of any fruit juice; 1 cup of milk; 1/2 cup of a regular soft drink; several pieces of hard candy; or 1 tablespoon of sugar or honey.

A client is advised to use a bulk-forming laxative to alleviate constipation. The nurse will recommend: mineral oil. milk of magnesia. Docusate (Colace). Psyllium (Metamucil).

Psyllium (Metamucil). Psyllium is a bulk-forming laxative. Docusate is a stool softener. Mineral oil is a lubricant. Both are laxatives, having milder action than cathartics, which include the stimulant milk of magnesia.

A nurse is planning care for a 59-year-old woman who is on ranitidine therapy. The nurse is concerned for the client's safety. What would be an appropriate nursing diagnosis? Potential Complication: Electrolyte Imbalance related to hypophosphatemia, secondary to drug therapy Acute Pain related to adverse drug effects, headache Diarrhea related to adverse effects of drug therapy Risk for Injury related to drug-induced somnolence, dizziness, confusion, or hallucinations

Risk for Injury related to drug-induced somnolence, dizziness, confusion, or hallucinations The appropriate nursing diagnosis related to safety would be Risk for Injury related to drug-induced somnolence, dizziness, confusion, or hallucinations. Diarrhea related to adverse effects of drug therapy and Acute Pain related to adverse drug effects, headache are appropriate nursing diagnosis for a patient taking ranitidine, but are not related to safety. Potential Complication: Electrolyte Imbalance related to hypophosphatemia, secondary to drug therapy would be appropriate for a client taking an aluminum hydroxide with magnesium hydroxide antacid (Maalox, Mylanta).

The nurse is participating in the care of a client in status epilepticus who is receiving phenobarbital IV. Which nursing diagnosis should the nurse prioritize during this client's current care? Acute confusion related to seizure activity and drug effects Risk for injury related to seizure activity Disturbed sensory perception related to adverse drug effects Disturbed thought processes related to seizure activity

Risk for injury related to seizure activity Seizures present an acute risk for injury which the nurse must prioritize when caring for the client. Cognitive and/or sensory disruptions are likely as well, but none of these presents such a high risk to the client's safety as the risk for injury.

Common, potentially serious, adverse effects of antibiotic drugs include: Hypopnea Skin rash Pain Constipation

Skin rash Explanation: Examine skin for any rash or lesions, examine injection sites for abscess formation, and note respiratory status—including rate, depth, and adventitious sounds to provide a baseline for determining adverse reactions. Report nausea, vomiting, diarrhea, skin rash, recurrence of symptoms for which the antibiotic drug was prescribed, or signs of new infection (e.g., fever, cough, sore mouth, drainage). These problems may indicate adverse effects of the drug, lack of therapeutic response to the drug, or another infection. Pain, constipation, and hypopnea are not common adverse effects of antibiotic drugs.

What assessment finding would signal the nurse to the possibility that the client's infusion of vancomycin is running too quickly? The client reports pain at the intravenous access site. The client is flushed and has a visible skin rash. The client is reporting nausea. The client's apical heart rate is irregular.

The client is flushed and has a visible skin rash. Explanation: It is very important to give IV infusions slowly, over 1 to 2 hours, to avoid an adverse reaction characterized by hypotension, flushing, and skin rash. This reaction, sometimes called red man syndrome, is attributed to histamine release. Irritation is likely the cause of pain at the IV access site. Neither cardiac arrhythmia nor nausea is associated with the rapid administration of this medication.

A client is given sulfasalazine to treat ulcerative colitis. The client begins to report flu-like symptoms including fever, cough, headache, and muscle aches and pains. The nurse notices red wheals on the client's face and neck. The nurse suspects which adverse effect? A)toxic epidermal necrolysis (TEN) B)aplastic anemia C)anaphylactic reaction D)Stevens-Johnson syndrome (SJS)

Stevens-Johnson syndrome (SJS) Explanation: Clients with SJS may report fever, cough, muscle aches and pains, and headache. These symptoms are accompanied by lesions on the skin and mucous membranes, eyes, and other organs. Lesions appear as red wheals or blisters, often starting on the face, in the mouth, or on the lips, neck, and extremities. TEN is a toxic skin reaction with sloughing of skin and mucous membranes. Anaphylactic reaction is a severe hypersensitivity reaction manifested by difficulty breathing, urticaria, itching, and possible respiratory arrest. Aplastic anemia is a blood disorder caused by damage to the bone marrow resulting in a marked reduction in the number of red blood cells and some white blood cells.

The pediatric nurse examines an 11-year-old client with mildly elevated blood pressure who is 15% above the upper age-based weight limits. What is the nurse's priority teaching point? Explain how to administer diuretics to maximize benefits and reduce adverse effects. Strategize with the client and family about ways to increase the child's activity level. Establish firm limits around the client's food intake. Explain the most common adverse effects of calcium channel blockers.

Strategize with the client and family about ways to increase the child's activity level. Treatment of childhood hypertension should be done very cautiously because the long-term effects of the antihypertensive agents are not known. Lifestyle changes should be instituted before drug therapy is started if at all possible. This should be done using a collaborative approach that includes the family. Weight loss and increased activity may bring an elevated blood pressure back to normal in many children. As a result, the priority teaching point is to help parents understand how to alter the child's diet to reduce weight and introduce family activities to increase exercise. A prescriptive approach to diet is likely to be perceived as punitive, possibly reducing adherence and self-esteem. Drug teaching would only be required if lifestyle changes is inadequate to lower blood pressure.

A client diagnosed with type 2 diabetes several months ago has presented for a scheduled follow-up appointment. Which stated behavior most clearly indicates that the client has established effective health maintenance? The client can describe the differences between type 1 and type 2 diabetes. The client exercises two to three times per week. The client reports having gone on a diet. The client frequently checks blood glucose levels.

The client frequently checks blood glucose levels. Explanation: Vigilant blood glucose monitoring is imperative in the management of diabetes. This shows effective health maintenance even more clearly than exercising. Dietary modifications must be undertaken with care in people with diabetes to avoid health consequences. Explaining pathophysiology does not necessarily show effective health maintenance.

A client with a complex medical history is showing signs and symptoms of sepsis. What aspect of this client's health history would rule out the safe and effective use of an aminoglycoside antibiotic? The client has chronic renal failure The client has a known latex allergy The client has a history of not adhering to treatment The client has type 2 diabetes, controlled with oral antihyperglycemics

The client has chronic renal failure Explanation: Renal failure would preclude the use of an aminoglycoside. Nonadherence must always be addressed, but this client variable is not specific to aminoglycosides. Neither latex allergies nor type 2 diabetes would necessarily rule out the use of an aminoglycoside.

The nurse in the emergency department receives a conscious client following a motor vehicle accident who has no known history of diabetes but whose blood glucose level is 325 mg/dL. What rationale does the nurse provide explaining this elevated blood glucose level? The client most likely ate a meal just before the accident. The client's stress reaction likely caused an increase in blood sugar. The client's accident was caused by undiagnosed hyperglycemia. The client may have sustained pancreatic trauma.

The client's stress reaction likely caused an increase in blood sugar. Explanation: The stress reaction elevates the blood glucose concentration above the normal range. In severe stress situations, the blood glucose level can be very high (300 to 400 mg/dL). The body uses that energy to fight the insult or flee from the stressor. It would be unlikely for a hyperglycemic episode to cause a change in consciousness that would result in an accident. Eating food does not cause such a large increase in glucose levels. Pancreatic trauma does not normally cause a precipitous increase in blood glucose levels.

A nurse obtains an allergy history from a client based on the understanding that which class is associated with a cross-sensitivity reaction with sulfonamides? Osmotic diuretics Carbonic anhydrase inhibitors Potassium-sparing diuretics Thiazide diuretics

Thiazide diuretics A cross-sensitivity reaction may occur with the thiazide diuretics and sulfonamides. For clients who take carbonic anhydrase inhibitors during treatment for glaucoma, contact the primary health care provider immediately if eye pain is not relieved or if it increases. When a client with epilepsy is being treated for seizures, a family member of the client should keep a record of all seizures witnessed and bring this to the primary health care provider at the time of the next visit. Contact the primary health care provider immediately if the number of seizures increases. Potassium-sparing diuretics can lead to hyperkalemia and is most likely to occur in clients with an inadequate fluid intake and urine output, those with diabetes or renal failure, older adults, and those who are severely ill, and teach the client to avoid the use of salt substitutes containing potassium. Osmotic diuretics such as mannitol or urea for treatment of increased intracranial pressure caused by cerebral edema, perform neurologic assessments (response of the pupils to light, level of consciousness, or response to a painful stimulus) in addition to vital signs at the time intervals ordered by the primary health care provider.

The nurse is caring for an older adult client who has been diagnosed with Parkinson disease and who will soon begin drug therapy. What nursing action will best promote safe and effective drug therapy? Teaching the family to best match medication doses to the daily severity of the client's symptoms Reassurance that many of the client's symptoms may be normal age-related changes rather than pathologic changes Detailed education about the need for outpatient blood pressure monitoring Thorough client and family teaching, supplemented with written materials

Thorough client and family teaching, supplemented with written materials Thorough and individualized client education is necessary to ensure safe and effective treatment. There is not normally a need for frequent blood pressure monitoring for most clients, and drug doses are not changed to address short-term symptoms. The symptoms of Parkinson's disease are not attributable to normal, age-related physiologic changes.

A client taking a chemical stimulant laxative and medications for heart failure and osteoarthritis calls the clinic and reports, "I'm just not feeling right." What is the priority question the nurse should ask this client? The amount of fiber intake Amount of fluid intake Timing of medication administration Previous effectiveness of laxatives

Timing of medication administration Because laxatives increase the motility of the gastrointestinal (GI) tract and some laxatives interfere with the timing or process of absorption, it is not advisable to take laxatives with other prescribed medications. The administration of laxatives and other medications should be separated by at least 30 minutes, so the nurse should question when the client is taking the laxatives and other medications. The other options may be questions the nurse would eventually ask, but the priority is timing of medication administration.

The nurse is caring for a client with a urinary tract infection caused by Escherichia coli and expects the health care provider to order which sulfonamide? A)Trimethoprim (TMP) and sulfamethoxazole (SMZ) B)Sulfisoxazole C)Sulfasalazine D)Sulfadiazine

Trimethoprim (TMP) and sulfamethoxazole (SMZ) Explanation: TMP and SMZ (Bactrim) are used to treat acute bacterial UTI, otitis media, and traveler's diarrhea that are due to Escherichia coli. The other medications are most effective against Haemophilus influenzae.

A client diagnosed with an H. pylori-associated ulcer has been prescribed treatment that includes two antimicrobials. What does the nurse identify as the reason for using multiple antimicrobials? A single antimicrobial is generally not strong enough to eradicate the infection. Two antimicrobials help prevent the emergence of drug-resistant H. pylori organisms. One antimicrobial tends to alter the action of the adjunct medications. Two different microbes are involved, requiring two different antimicrobials.

Two antimicrobials help prevent the emergence of drug-resistant H. pylori organisms. Explanation: Effective treatment of an H. pylori-associated ulcer involves two antimicrobials—including amoxicillin, clarithromycin, metronidazole, or tetracycline—to reduce the risk of resistance. None of the other options present accurate information regarding the basis for treatment.

The nurse is caring for a client who has been prescribed a sulfonamide but does not have an infection. The nurse is aware that the medication has been prescribed to treat what condition? Gastritis Crystalluria Stomatitis Ulcerative Colitis

Ulcerative Colitis Explanation: Sulfonamides are primarily prescribed for treatment of infections but they are also used to manage ulcerative colitis. Stomatitis and crystalluria are possible adverse effects of sulfonamides. Gastritis is an inflammation of the stomach and sulfonamides would not be used to treat this condition.

The nurse is giving discharge instructions to a client with an upper respiratory infection who has been advised to take an over-the-counter (OTC) topical nasal decongestant. What should the nurse teach the client in order to prevent rhinitis medicamentosa? Use the medication for as short a time as possible. Increase fluid intake for the duration of treatment. Avoid using antihistamines at the same time as the decongestant. Avoid using nonsteroidal anti-inflammatory drugs (NSAIDs) concurrently.

Use the medication for as short a time as possible. An adverse effect that accompanies frequent or prolonged use of topical nasal decongestants is rebound congestion, technically called rhinitis medicamentosa. Avoiding the overuse of the medication is a key strategy for preventing this complication. Antihistamines do not cause it, and use of NSAIDs is not problematic. Increasing fluid intake helps with relieving congestion but does not prevent rhinitis medicamentosa.

Rifaximin has been prescribed for a client diagnosed with traveler's diarrhea. What information should the nurse include in the teaching plan about the medication? Used for E. coli infections. The diarrhea is usually associated with fever and bloody stools. Adverse effects include migraine, joint pain, and nausea. Superinfections will not occur with this medication.

Used for E. coli infections. Rifaximin is a structural analog of rifampin. It was developed specifically to treat travelers' diarrhea due to noninvasive E. coli in clients older than 12 years of age. Use for diarrhea in the presence of fever or bloody stools is not warranted. Superinfection may occur, requiring termination of rifaximin. Adverse effects include flatulence, headache, stomach pain, urgent bowel movements, nausea, constipation, fever, vomiting, and dizziness.

A client is in shock and is receiving dopamine. The nurse knows that the proper way dopamine should be given is by what method? Via an electric infusion pump, by itself, and titrated to desired systolic blood pressure On an electric infusion pump, through the main IV line with other medications, and titrated to the desired heart rate IV bolus through the IV closest to the heart, followed by a normal saline flush Via an IM injection diluted with normal saline

Via an electric infusion pump, by itself, and titrated to desired systolic blood pressure Dopamine should be administered via an electric infusion pump, by itself and titrated to the desired systolic blood pressure. It is not an IM medication. It should not be given with other medications, and it is not given as an IV bolus, no matter where the IV is located.

The nursing instructor is teaching about anti-infectives and informs students that viruses have been difficult to treat in the past. He indicates the reason for this to be which of the following? Viruses cause superinfections that make treatment more difficult. Viruses have the ability to change because they use DNA and RNA of other cells. Viruses do not use host cells to grow and divide. Viruses have a typical cell structure that is hard to treat.

Viruses have the ability to change because they use DNA and RNA of other cells. Explanation: When teaching about viruses, the instructor should inform the students that viruses do not have a typical cell structure and use host cells to grow and divide. Because viruses use the DNA and RNA of other cells, their ability to change has posed a problem in making drugs to treat viral infections.

A homeless client presented at a street clinic with reports of hemoptysis and subsequently tested positive for TB. Which characteristics of the client's present circumstances would preclude treatment with INH? lacks the social support necessary to promote adherence to treatment a type 2 diabetic who practices poor glycemic control an alcoholic who has hepatitis C (HCV) treated with IV vancomycin earlier this year for a bone infection

an alcoholic who has hepatitis C (HCV) Because of the high potential for hepatotoxicity that is associated with INH, its use is likely contraindicated in a client with alcoholism and HCV. INH must be used with caution in those with diabetes, but the drug is not necessarily contraindicated. Previous antibiotic use is not a confounding factor in treatment and lack of social support would need to be addressed, but does not preclude treatment.

What nursing actions should be taken for a client prescribed medication that is expected to block the sympathetic nervous system? Select all that apply. implementing interventions to minimize risks associated with dizziness assessment of physiological response to sexual function monitoring for tachycardia monitoring for presence of urinary urgency administering a PRN anti-diarrhea medication

assessment of physiological response to sexual function monitoring for presence of urinary urgency administering a PRN anti-diarrhea medication implementing interventions to minimize risks associated with dizziness Sexual dysfunction, increased urine flow, increased GI motility, and orthostatic hypotension are all possible side effects of adrenergic blockage. Bradycardia rather than tachycardia should be monitored for.

A client, newly diagnosed with Parkinson's disease, has been prescribed an anticholinergic drug. What assessment should the nurse perform when monitoring for adverse effects of anticholinergic agents used for the treatment of Parkinson disease? focused respiratory assessment monitoring of prothrombin time and international normalized ratio (INR) assessment of the client's bowel pattern monitoring of laboratory results for blood dyscrasias

assessment of the client's bowel pattern Anticholinergics often cause decreased bowel motility, resulting in constipation. This is especially true in older adults. Anticholinergics are not associated with coagulation disorders, blood dyscrasias, or respiratory problems.

A client being treated with an oral penicillin should be encouraged to administer the medication on which schedule to best achieve a therapeutic effect? Select all that apply. with meals at bedtime around the clock upon rising in the am at regular intervals

at regular intervals around the clock Explanation: Clients should aim to take penicillins at even intervals, preferably around the clock. These drugs are not normally taken with food.

When describing the action of fluoroquinolones to treat infection to a group of nursing students, which would the instructor include? bacteriostatic fungicidal fungistatic bactericidal

bactericidal Explanation: The fluoroquinolones exert bactericidal effects by interfering with the synthesis of bacterial DNA. This interference prevents cell reproduction, causing death of the bacterial cell. They do not inhibit the growth as in bacteriostatic. These group of drugs are not used to treat or reduce fungal growth or infections.

In providing care to a client with wheezing, the nurse knows that a medication targets which receptors? alpha 1 beta 1 alpha 2 beta 2

beta 2 Beta 2 receptors are found on the bronchial smooth muscles, and medications that stimulate beta 2 receptors cause bronchodilation. Alpha 1 receptors cause vasoconstriction of peripheral blood vessels. Alpha 2 receptors decrease tone, motility, and secretions of the GI tract. Beta 1 receptors increase heart rate and force of myocardial contraction.

The nurse is assessing the client for adverse CNS effects after administering a nonselective adrenergic blocker based on the understanding that these effects occur due to: loss of vascular tone. blockage of norepinephrine's effect. increased parasympathetic dominance. vasodilation of the mucus membrane vessels.

blockage of norepinephrine's effect. CNS effects occur with nonselective adrenergic blockers because the effects of norepinephrine are blocked in the CNS. GI effects of a nonselective adrenergic blocker are related to parasympathetic dominance. Cardiovascular effects occur with nonselective adrenergic blockers because of the lack of stimulatory effects and loss of vascular tone in the cardiovascular system. Respiratory effects occur with nonselective adrenergic blockers because of the loss of bronchodilation and vasodilation of the mucous membrane vessels.

The concentration of any drug in the body is referred to as which of the following? percentage degree ratio blood level

blood level Explanation: The concentration of any drug in the body is referred to as the blood level. An inadequate concentration (or inadequate blood level) of penicillin may produce bacteriostatic activity, which may or may not control the infection.

A client with a gram-negative bacterial infection has been prescribed a fluoroquinolone. During the preadministration assessment, the nurse should ensure that which laboratory tests, if prescribed, are obtained before the first dose of fluoroquinolones is administered to the client? renal and hepatic function tests culture and sensitivity test complete blood count urinalysis

culture and sensitivity test Explanation: The nurse should ensure that culture and sensitivity tests are done before the client receives the first dose of fluoroquinolones. The nurse should ensure that renal and hepatic function tests, urinalysis, and tests to determine blood count are conducted if the client is to be administered aminoglycosides.

When fluoroquinolones are administered intravenously, the nurse would check the needle site and area around the needle site for extravasation at which frequency? every 2 hours every hour every 4 hours every 6 hours

every hour Explanation: When fluoroquinolones are administered intravenously, the nurse would check the needle site and area around the needle site for extravasation—escape of fluid from a blood vessel into surrounding tissue, every hour. Times longer than every hour can lead to necrosis of the tissue if extravasation occurs.

A client has been prescribed medication therapy for the treatment of newly diagnosed asthma. During teaching, the nurse should alert the client to potential exacerbation of what concurrent medical condition? urinary retention dermatitis gastroesophageal reflux disease cataracts

gastroesophageal reflux disease Asthma may aggravate gastroesophageal reflux disease because antiasthma medications that dilate the airways also relax muscle tone in the gastroesophageal sphincter and may increase acid reflux. The relationship between asthma medications and the other options is not supported by research data.

A nurse instructs an older adult client to eat within 30 minutes of the administration of which antidiabetic drugs to prevent hypoglycemia? glipizide pioglitazone acarbose metformin

glipizide Explanation: A nurse should advise a client, especially an older adult client, to eat within 30 minutes of the administration of sulfonylureas like glipizide to prevent hypoglycemia. Pioglitazone is for type 2 diabetes and is administered three times a day. Metformin is for type 2 diabetes and the client may be instructed to take it two to three times a day with meals. The alpha-glucosidase (α-glucosidase) inhibitor acarbose prevents the after meal surge in blood glucose by delaying the digestion of carbohydrates and absorption of carbohydrates in the intestine and is taken three times a day.

A nurse is caring for a client in the critical care unit. Phentolamine (Regitine, OraVerse) has been ordered for the management of tissue necrosis caused by extravasation of parenterally administered drugs. Before administering this drug, the nurse will check the client's chart for indications of: obesity. diabetes mellitus. peptic ulcer disease. history of acute myocardial infarction.

history of acute myocardial infarction. The nurse will check the chart for any indication of a history of acute myocardial infarction. Phentolamine is contraindicated in clients with this history or any evidence of coronary artery disease because of its cardiac-stimulating effects and resultant increases in myocardial oxygen demand. This drug is used with caution in clients with gastric and duodenal ulcers because the drug has a histamine-like effect. Obesity and diabetes mellitus should not be a factor with the use of phentolamine.

A 66-year-old woman with a history of recurrent urinary tract infections has been admitted to the hospital with a diagnosis of pyelonephritis. The client began treatment 36 hours ago with intravenous gentamicin and is being monitored closely. The nurse is aware that the therapeutic effect of the client's drug regimen is primarily a result of: enhancement of the client's immune function. alterations in the osmolarity of bacterial cytoplasm. lysis of the bacterial cell walls. interference with bacterial reproduction.

interference with bacterial reproduction. Explanation: Gentamicin exerts its effect by entering the bacterial cell and binding to the 30S ribosomal subunit. This event leads to a misreading of the information used within the cell to form proteins. This leads indirectly to cell wall rupture. Gentamicin does not enhance immune function or change the osmolarity of bacterial cytoplasm.

A client is newly diagnosed with Parkinson disease, and levodopa has been prescribed. What health education should the nurse provide? the fact that adverse effects will subside when the body adjusts to the drug matching doses to the daily severity of symptoms identifying foods and supplements high in vitamin B6 the need for weekly laboratory blood work for the first 6 weeks

identifying foods and supplements high in vitamin B6 The priority information would be to avoid vitamin B6 intake, which would include grains and bran. Vitamin B6 speeds the conversion of levodopa to dopamine before it can cross the blood-brain barrier. This leads to Parkinson symptoms. Weekly blood work is not required, and adverse effects are not necessarily self-limiting. Doses are taken as scheduled, not adjusted on the short-term basis of symptoms.

The nurse is caring for an obese, adult client with little social support who was just diagnosed with type 2 diabetes. When developing this client's plan of care, what is the priority nursing diagnosis? ineffective coping disturbed sensory perception risk for infection imbalanced nutrition: more than body requirements

imbalanced nutrition: more than body requirements Explanation: The priority with this client is to address their obesity because losing weight may eliminate the problem. The client may need support to cope with the diagnosis but these are not the priority concern. Disturbed sensory perception is not indicated to be a problem at this time. Diabetes creates a risk for infection, but the client's obesity is a priority.

The nurse is assessing a client who was prescribed levodopa 1 week earlier. What change in the client's status would suggest the effectiveness of drug therapy? improved attention span improvement in handwriting stability of mood slower chewing of food

improvement in handwriting The nurse would evaluate the client for improvement in function and reduction in symptoms. With preparations containing levodopa and with dopaminergic agents, assess for improvement in mobility, balance, posture, gait, speech, handwriting, and ability to provide self-care. An improvement in mood may suggest successful treatment. Slower chewing would suggest worsening motor deficits. Attention span is unlikely to be affected.

A male client newly diagnosed with tuberculosis (TB) asks the nurse if medications will make him better. The nurse informs the client that sometimes treatment fails and why this happens. What are some reasons for treatment failure? (Select all that apply.) lack of vegetables in person's diet lack of access to vitamins noncompliance with the therapeutic regimen type of facility where the client resides inadequate initial drug treatment

inadequate initial drug treatment noncompliance with the therapeutic regimen At times, treatment fails because of inadequate initial drug treatment or noncompliance with the therapeutic regimen. Retreatment usually includes the use of four or more antitubercular drugs.

The stimulation of beta1 adrenergic receptors in the heart by epinephrine would cause what result? vasoconstriction of the coronary arteries increased heart rate decreased heart muscle contractions decreased oxygen demand by the myocardium

increased heart rate The predominant effect in response to activation of beta1 receptors in the heart is cardiac stimulation. Beta1 activation results in increased force of myocardial contraction, or a positive inotropic effect and increased speed of electrical conduction in the heart.

What is the expected outcome when phenylephrine is administered? Select all that apply. decreased renal perfusion increased diastolic blood pressure increased cardiac output decreased peripheral vascular resistance increased systolic blood pressure

increased systolic blood pressure increased diastolic blood pressure decreased renal perfusion Phenylephrine is a synthetic adrenergic drug that stimulates alpha-adrenergic receptors to produce vasoconstriction. As a result, it constricts arterioles and raises systolic and diastolic blood pressures. Vasoconstriction decreases cardiac output and renal perfusion and increases peripheral vascular resistance and blood pressure.

When describing the action of sulfonamides to a client, the nurse integrates understanding of the drug's action with effects exerted due to which mechanism? inhibition of bacterial RNA-dependent protein synthesis disruption of bacterial cell wall synthesis inhibition of DNA-gyrase inhibition of the activity of folic acid in bacterial cell metabolism SUBMIT ANSWER

inhibition of the activity of folic acid in bacterial cell metabolism Explanation: Sulfonamides exert their effects by inhibiting the activity of folic acid in bacterial cell metabolism. Sulfonamides do not inhibit DNA, RNA, or cell wall synthesis.

A nursing instructor is preparing a teaching plan for a nursing pharmacology class on the action of fluoroquinolones. Which would the instructor include? interfering with protein synthesis disrupting the bacterial cell wall interfering with DNA synthesis in the bacterial cell blocking ribosomal reading of mRNA

interfering with DNA synthesis in the bacterial cell Explanation: The fluoroquinolones exert their bactericidal effect by interfering with the synthesis of bacterial DNA by not allowing the cell to reproduce. The tetracyclines are bacteriostatic and exert their effect by inhibiting bacterial protein synthesis. Penicillins act to disrupt the bacterial cell wall. The aminoglycosides exert their bactericidal effect by blocking the ribosome from reading the mRNA, a step in protein synthesis necessary for bacterial multiplication.

The nurse recognizes that what drug classification increases the risk of ototoxicity and nephrotoxicity when prescribed with gentamicin? proton pump inhibitors loop diuretics tricyclic antidepressants benzodiazepines

loop diuretics Explanation: Loop diuretics given simultaneously with gentamicin increase the risk of nephrotoxicity by decreasing fluid volume, thereby increasing drug concentrations in serum and tissues. Loop diuretics may also contribute to ototoxicity. This risk is not associated with therapies that include both gentamicin and any of the other options.

A client is prescribed an antidiarrheal that acts directly on the muscle wall of the bowel to slow motility. The nurse would identify which drug as being prescribed? omeprazole sodium bicarbonate loperamide diphenoxylate

loperamide Explanation: Loperamide is not chemically related to opioid drugs and treats diarrhea by acting directly on the muscle wall of the bowel to slow motility. Diphenoxylate is chemically related to opioid drugs and treats diarrhea by decreasing intestinal peristalsis. Sodium bicarbonate is used to reduce stomach acid levels and should be used only temporarily. Omeprazole is a proton pump inhibitor that relieves symptoms of GERD and other stomach conditions.

A clinic nurse has been assigned a 49-year-old female client who has a history of diabetes. A recent diagnosis of hypertension has been made, and the client has been prescribed a thiazide diuretic and labetalol. The client will be scheduled to return to the clinic once a month for the next 6 months. A priority action by the nurse will be to: weigh the client. question the client about her dietary intake. monitor the client's respiratory rate. monitor the client's blood pressure.

monitor the client's blood pressure. Monitoring of blood pressure would be the priority assessment in the care of this client. Questioning the client about her dietary intake and weighing her would be appropriate, but secondary in importance. It is unnecessary to monitor the client's respiratory rate.

A client, newly diagnosed with a seizure disorder, has been prescribed valproic acid. When assessing for adverse effects, what assessment should the nurse prioritize? performing musculoskeletal assessment monitoring the client's liver enzyme levels assessing the client's apical heart rate and rhythm assessing the client for signs of esophageal varices

monitoring the client's liver enzyme levels Valproic acid is associated with liver toxicity., so the nurse should periodically check the client's liver enzyme levels. Esophageal varices, cardiac damage and muscle weakness are not associated with valproic acid therapy.

The nurse realizes that vancomycin is a powerful drug and can treat serious gram-positive infections that do not respond to treatment with other anti-infectives. Serious adverse effects classic for vancomycin include which of the following? (Select all that apply.) increased risk for bleeding increased risk for hyperglycemia nephrotoxicity ototoxicity

nephrotoxicity ototoxicity Explanation: Nephrotoxicity (damage to the kidneys) and ototoxicity (damage to the organs of hearing) may occur with administration of vancomycin. Increased bleeding and hyperglycemia are not known adverse effects of this drug.

The nurse is interviewing a client with a history of diabetes mellitus, obesity, and a recent myocardial infarction. What relevant assessment data should the nurse discuss with the client associated with coronary artery disease (CAD)? Select all that apply. typical A1C results cholesterol consumption nicotine use experience of angina pain history of insomnia

nicotine use typical A1C results cholesterol consumption experience of angina pain Increased cholesterol consumption, nicotine use, poor glucose control, and the presence of episodes of angina pain are all relevant to the diagnosis of coronary artery disease. While an appropriate assessment topic, insomnia is not directly associated with CAD.

The client has been started on nitroglycerin ointment for angina. The nurse identifies that the nitroglycerin ointment has been effective if the client reports: that he gets a headache each time the ointment is applied. dizziness after each application. no episodes of angina since ointment was initiated. minimal episodes of angina.

no episodes of angina since ointment was initiated. Topical nitroglycerin is used to manage angina. The treatment is effective if the client reports no episodes of angina. Headache and dizziness are adverse effects of the medication.

A female client is being treated for Mycoplasma pneumoniae pneumonia. She is allergic to penicillin and is being given azithromycin (Zithromax) in capsule form. The nurse will inform the client that she will need to take the capsule immediately after she eats. on an empty stomach. with food. with or without food.

on an empty stomach. Explanation: Because the absorption of azithromycin capsules is decreased in the presence of food, azithromycin capsules should be taken on an empty stomach rather than after a meal. Unlike azithromycin capsules, azithromycin tablets have an increased absorption when given with a meal with high fat content and may be given with or without food.

The nurse frequently sees mature male clients with BPH seeking treatment for symptoms. After ensuring they do not test positive for prostate cancer, pharmacological intervention is one of a few treatment modalities available. Pharmacokinetics indicate alpha1-blocking agents as a treatment choice due to their: insulin secretion suppression. activation of reflexes, which decreases blood pressure. tendency to decrease urine flow. prevention of smooth muscle contraction in nonvascular tissues.

prevention of smooth muscle contraction in nonvascular tissues. Alpha1-adrenergic blocking drugs can prevent alpha-mediated contraction of smooth muscle in nonvascular tissues.

A 23-year-old client presents at the clinic with a migraine headache. What beta-adrenergic blocking agent will best prevent future migraine headaches? sotalol nadolol timolol propranolol

propranolol Propranolol is indicated for the treatment of hypertension, angina pectoris, idiopathic hypertrophic subaortic stenosis, supraventricular tachycardia, tremor; prevention of reinfarction after myocardial infarction; adjunctive therapy in pheochromocytoma; prophylaxis of migraine headache; and management of situational anxiety. The other options do not treat or prevent migraine headaches.

A client with a diagnosis of rhinosinusitis has been prescribed ciprofloxacin 250 mg SC b.i.d. When contacting the prescriber, the nurse should question the: choice of drug. dose. frequency. route.

route Recommended parameters for ciprofloxacin are 100-500 mg b.i.d. PO for up to 6 weeks. As a result, the nurse should have the provider confirm the correct route.

The client has a seizure that causes rhythmic twitching of the left hand for 90 seconds with no loss of consciousness and then stops. The nurse notes this same action repeated many times throughout the day and documents this as what type of seizure? simple partial seizure Jacksonian seizure myoclonic seizure psychomotor seizure

simple partial seizure This client is having simple partial seizures, which occur in a single area of the brain and may involve a single muscle movement or sensory alteration. Myoclonic seizures involve short, sporadic periods of muscle contractions that last for several minutes. Jacksonian seizures begin in one area of the brain and involve one part of the body and then progressively spread to other parts of the body; they can develop into generalized tonic-clonic seizures. Psychomotor seizures are complex seizures that involve sensory, motor, and psychic components.

A nurse is speaking to a 62-year-old female client who has been started on sulfisoxazole, a sulfonamide antibiotic. The nurse should teach this client to contact the health care provider if the client experiences what adverse effect associated with the drug? tachycardia polydipsia dizziness skin rash or itching

skin rash or itching Explanation: Clients taking sulfonamides should be instructed to contact the prescriber if they experience skin rash or itching. These symptoms may indicate a sulfonamide-induced allergic reaction and the need to change or stop the drug.

How does histamine release contribute to ulcer formation? initiates pepsin synthesis stimulates denaturing of gastrin destroys parietal cells stimulates gastric acid secretion

stimulates gastric acid secretion Explanation: Vagal stimulation causes release of histamine from cells in the gastric mucosa. The histamine then acts on receptors located on the parietal cells to increase production of hydrochloric acid, which is a factor in stomach ulcer formation. Histamine is not associated with either pepsin or gastrin, and it does not destroy the parietal cells.

A client who has been on long-term Geocillin therapy for repeated urinary tract infections (UTIs), reports severe abdominal cramping. The client also states that he has been having bloody diarrhea and rectal bleeding. What should the nurse suspect? adverse reaction side effect superinfection anaphylaxis

superinfection Explanation: A superinfection can develop rapidly and is potentially life-threatening. Bacterial superinfections are seen with administration of the oral penicillins and occur in the bowel. Symptoms include bloody diarrhea, rectal bleeding, fever, and abdominal cramping. The others are reactions to penicillin but do not have these specific symptoms. Reference:

What is the treatment goal for a preoperative prescription for neomycin? enhancement of healthy intestinal flora suppression of intestinal bacteria to produce ammonia lost through the administration of anesthesia prevention of postoperative wound infections

suppression of intestinal bacteria Explanation: Neomycin and kanamycin may be given before bowel surgery to suppress intestinal bacteria. These drugs are not used to produce ammonia, enhance normal flora, or prevent postoperative wound infections.

A client is prescribed phenytoin and reports gastrointestinal (GI) upset. The nurse should tell the client to: take the medication with meals. take the medication at bedtime. take the medication two hours after ingesting dairy products. ask the health care provider to change the route from oral to injection.

take the medication with meals. When phenytoin is taken orally, the drug should be taken with meals to avoid GI upset, rather than two hours after dairy products. Phenytoin by injection is a more invasive route. First, the client can be instructed to take the medication with meals. Taking the medication at bedtime is not as effective in preventing GI upset than taking the drug with meals.

The client is admitted to the acute care facility with acute septicemia and has orders to receive gentamicin and ampicillin IV. The nurse is performing an admission assessment that includes a complete nursing history. What information provided by the client would indicate the need to consult the healthcare provider before administering the ordered medication? takes high-dose furosemide daily had prostate surgery 3 months ago history of hypothyroidism allergy to peanuts and peanut products

takes high-dose furosemide daily Explanation: Aminoglycosides should be avoided if the client takes a potent diuretic because of the increased risk of ototoxicity, nephrotoxicity, and neurotoxicity. Learning the client takes a potent diuretic would indicate the need to consult with the healthcare provider before administering gentamicin. Prostate surgery, hypothyroidism, and an allergy to peanuts would not preclude administration of these medications and would not indicate a need to consult with the provider.

A client with a deep wound producing large amounts of purulent drainage is prescribed an antibiotic. The nurse should express concern about the potential effectiveness of the treatment if which medication is prescribed? A)penicillin B)trimethoprim-sulfamethoxazole C)tetracycline D)doxycycline

trimethoprim-sulfamethoxazole Explanation: Sulfonamides like trimethoprim-sulfamethoxazole halt multiplication of new bacteria but do not kill mature, fully formed bacteria. Except for the topical sulfonamides used in burn therapy, the presence of pus, serum, or necrotic tissue interferes with sulfonamide action because these materials contain PABA. Trimethoprim-sulfamethoxazole would not be the appropriate medication to manage this client's needs. The remaining options are acceptable medications for this application.

The teaching priority for a diabetic male client being treated with a non-specific beta-blocker would be to: understand signs and symptoms of hypo- or hyperglycemic reaction. take his own pulse. avoid smoke-filled rooms. weigh himself once a week at the same time of day.

understand signs and symptoms of hypo- or hyperglycemic reaction. Because the beta-blockers stop the signs and symptoms of a sympathetic stress reaction, and the signs and symptoms associated with hypo- or hyperglycemia, the diabetic client taking a beta-blocker will need to learn new signs and symptoms of these reactions. Taking his pulse, weekly weighing, and avoiding smoke-filled rooms are good health practices and should be done, but are not specifically needed by a diabetic client taking a beta-blocker.

The nurse is caring for a client who is receiving amikacin for the treatment of a serious Staphylococcus aureus infection. What assessment should the nurse prioritize? cardiac monitoring urine output and creatinine clearance skin assessment and monitoring of liver enzyme levels vision assessment

urine output and creatinine clearance Explanation: Aminoglycosides such as amikin come with a black box warning alerting healthcare professionals to the serious risk of nephrotoxicity. Visual alterations are not usually reported in relation to this drug. Similarly, cardiac and liver function are not usually impacted by this drug.

A male client is to begin glyburide (DiaBeta) for type 2 diabetes. Before the drug therapy begins, a priority action by the nurse will be to assess the client's: blood pressure. potassium level. use of salt in his diet. use of alcohol.

use of alcohol. Explanation: The nurse should assess the client's alcohol consumption before therapy is initiated. Concomitant alcohol use increases the rate of glyburide metabolism and may cause a disulfiram-like reaction. Even though the client's blood pressure should be assessed, it is not as important as making sure that the client is aware of the serious reaction that alcohol can cause while on glyburide. The amount of salt used in the patient's diet is important as well as the client's potassium level, but not specifically in relation to the use of glyburide.


संबंधित स्टडी सेट्स

Chapter 17. Activity - Based costing and analysis

View Set

Major Nerve Plexuses and Select Nerves

View Set

Almost, Maine - Daniel (Man) and Hope

View Set

Communities Develop and Montana Becomes a Territory

View Set

A&P Ch. 12: Membrane Potential and the Action Potential

View Set

chapter 1: dynamic environment of hrm

View Set